Está en la página 1de 96

The number of undergraduate degrees in engineering awarded by colleges and universities in the United

States increased by more than twice from 1978 to 1985.

(A) increased by more than twice


(B) increased more than two times
(C) more than doubled
(D) was more than doubled
(E) had more than doubled.

A is just idiomatically incorrect. I have marked A to this question twice on my


GMAT Prep and that is why I have this question in my log. Just learn that the
construction in A is unidiomatic. Yes, we really can't do anything about it :/
-------------------------------------------------------------------The state religion of ancient China, which came into existence at least as far back as the Han Dynasty (206 B.C. to
A.D. 220) and perhaps as early as the Western Zhou Dynasty (1050 B.C. to 771 B.C.), allowed only emperors
performing the ritual worship of heaven, perceiving them as special links between the earthly and celestial
realms.
A. only emperors performing the ritual worship of heaven, perceiving them as
B. only emperors to perform the ritual worship of heaven, with the perception of them being
C. the ritual worship of heaven to be performed only by emperors, who were perceived as
D. the ritual worship of heaven, performed by emperors only, with the perception of them as
E. the ritual worship of heaven as performed by emperors only, who were perceived to be
2. Meaning change with the use of 'only emperors'. Read C and you'll know the difference.
1. Allow can be used with 'to' and 'for' i.e. Allow X to do Y OR Allow for. This is used incorrectly here.
-------------------------------------------------------------------------------------------------Regarding Subordinate/Substantive Clause, there is a very good article written by Garry. You can read it here. I have
this already in my error log :)
http://magoosh.com/gmat/2012/substantive-clauses-on-the-gmat/
----------------------------------------------------------------------------------------------------------Almost like clones in their similarity to one another, the cheetah species homogeneity makes
them especially vulnerable to disease.
A. the cheetah species homogeneity makes them especially vulnerable to disease
B. the cheetah species is especially vulnerable to disease because of its homogeneity
C. the homogeneity of the cheetah species makes it especially vulnerable to disease
D. homogeneity makes members of the cheetah species especially vulnerable to disease
E. members of the cheetah species are especially vulnerable to disease because of their homogeneity
OE E
Why not B? Isn't clones being compared with the cheetah species???
We use "one another" to denote a reciprocal relationship among members of a group. For example, "the teammates
hugged one another," is appropriate usage - teammates are engaged in an activity with other teammates - but "the
team hugged one another" is not appropriate, as "team" is singular, and thus there is no other entity for the "team" to
interact with.

In E, we have "Almost like clones in their similarity to one another, members of the cheetah species are especially
vulnerable to disease because of their homogeneity." "One another" is used appropriately here. It is part of an
introductory modifier referring to "Members."
In B, we have the "Almost like clones in their similarity to one another, the cheetah species is especially vulnerable to
disease because of its homogeneity."
Here we have one species, and there is no other entity for that species to be similar to.

---------------------------------------------------------------------------------------------------Magossh:
Require that X be Y
Correct: The new regulations require that prospective employees be subjected to rigorous screening.
Incorrect: The new regulations require prospective employees to be subjected to rigorous screening.

Estimate to be
Correct: The fossils are estimated to be more than 65 million years old.
Incorrect: The fossils are estimated as more than 65 million years old.

Prohibit X from Y
Correct: The ban will prohibit those without adequate documentation from purchasing guns.
Incorrect: The ban will prohibit those without adequate documentation to purchase handguns.

Believe X to be Y
Correct: Astrophysicists believe the recent disturbances in radio transmissions to be a result of solar flares.
Incorrect: Astrophysicists believe that the recent disturbances in radio transmissions to be a result of solar flares.

Consider X Y (no to be)


Correct: Most musicologists consider Joseph Haydn the father of the sonata.
Incorrect: Most musicologists consider Joseph Haydn to be the father of the sonata.
Incorrect: Most musicologists consider Joseph Haydn as the father of the sonata.

X expected to Y
Correct: Tax rates are expected to increase next year.
Incorrect: Tax rates are expected to be increasing next year.
Not onlybut also
Correct: Idioms are not only difficult to memorize but are also easy to mix up.
Incorrect: Idioms are not only difficult to memorize but are easy to mix up.

Neithernor

Correct: Studies show that neither studying alone nor in groups is optimal.
Incorrect: Studies show that neither studying alone or in groups is optimal.
Just asso too
Correct: Just as caffeine can boost arousal so too can vigorous walking.
Incorrect: Just as caffeine can boost arousal vigorous walking can also.

Prefer X to Y
Correct: The blue macaw prefers lush tropical habitats to the dry climate found in the southeastern part of Brazil.
Incorrect: The blue macaw prefers lush tropical habitats over the dry climate found in the southeastern part of Brazil.
-----------------------------------------------------------------------

Never before had taxpayers confronted so many changes at once as they had in the Tax Reform Act of 1986.
A. so many changes at once as they had in
B. at once as many changes as
C. at once as many changes that there were with
D. as many changes at once as they confronted in
E. so many changes at once that confronted them in
Also in A, "had" is incorrect,it should be "did"
Never before had taxpayers confronted so many changes at once as they had in the Tax Reform Act of 1986.
Never before had taxpayers confronted so many changes at once as they did in the Tax Reform Act of 1986
---------------------------------

SUCH THAT :
RON:
* "such that" doesn't make sense.
in fact, i don't think you should pick a sentence with "such that" in one piece. if anyone can prove me wrong on this,
please do.
the ONLY non-awkward, correct sentences i've seen with "such that" are sentences about math or physics (e.g.,
"choose x and y such that x + y = 10").
WHILE
"such NOUN that" is a perfectly respectable construction in formal english.

---------------------------------Usage of ''lower'':
"Lower" is the comparative of "low," so it should only be applied to things that could be described as low.
The average temperature is lower in the winter than in the summer.
Lower the water level in the pool by pulling the drain plug.
Lower can work with both countable and uncountable noun.

-------------------------------

Oberlin College in Ohio was a renegade institution at its 1833 founding for deciding to accept both men and
women as students.
(A) at its 1833 founding for deciding to accept
(B) for the decision at its 1833 founding to accept
(C) when it was founded in 1833 for its decision to accept
(D) in deciding at its founding in 1833 to accept
(E) by deciding at its founding in 1833 on the acceptance of
A, B can be ruled out because of the wrong usage of "1983 fundin".
Correct phrase would be " funding in 1983".
E is horrible.it tries to convert a verb(accept) into a noun ( the acceptance of)
The meaning is not clear in C.
I could infer 2 meanings:
It was a renegade institute. When it was founded
Or
It was a renegade institute for its decision to accept.....
I was not able to club both the meanings.D removes all these errors
------------------------------------More than 300 rivers drain into Siberia's Lake Baikal, which holds 20 percent of the world's fresh water, more than all
the North American Great Lakes combined.
(A) More than 300 rivers drain into Siberia's Lake Baikal, which holds 20 percent of the world's fresh water, more than
all the North American Great Lakes combined.
(B) With 20 percent of the world's fresh water, that is more than all the North American Great Lakes combined,
Siberia's Lake Baikal has more than 300 rivers that drain into it.
(C) Siberia's Lake Baikal, with more than 300 rivers draining into it, it holds more of the world's fresh water than all
that of the North American Great Lakes combined, 20 percent.
(D) While more than 300 rivers drain into it, Siberia's Lake Baikal holds 20 percent of the world's fresh water, which is
more than all the North American Great Lakes combined.
(E) More than all the North American Great Lakes combined, Siberia's Lake Baikal, with more than 300 rivers
draining into it, holds 20 percent of the world's fresh water.
I had a hard time understanding this question. Especially the function of this modifier:"more than all the North
American Great Lakes combined".
Posting from my error log:
more than all is an absolute modifier or a noun + noun modifier; these modifiers have considerable flexibility, and
can be taken to refer either to a noun or to an entire clause.
more is a noun here and than is a comparative conjuction. So a noun+ noun modifier construction.
neither "lake" nor "water" would be a legitimate referent for this modifier -- you couldn't say "the lake is more than..." ,
but neither could you say "the water is more than..."
this sentence will only make sense if the modifier is allowed to modify the preceding clause, which talks about holding
water (because "holding more" actually makes sense).

--------------------------------------------------------------------------Having Modifier:

"Having seen the mess that George W. Bush made of his war on terror, especially in Iraq, Mr Obama is understandably wary".

In 1988, the council on Economic Priorities began publishing Shopping for a Better World, with the simple thesis
of consumers having the power to change companies by the simple expedient of refusing to buy.
A. with the simple thesis of consumers having
B. which had the simple thesis of consumers having
C. where the thesis was simple: consumers having
D. with a thesis that is a simple one: consumers have
E. whose thesis was simple: consumers have
Let's have a look at options A and B
In options (A) and (B), "having the power to change companies....." is an -ing modifier that's not followed by a
comma. Therefore, it's a modifier that serves as an adjective, modifying the immediately preceding noun "consumers"
The problem is since it's a modifier - can be omitted from the basic structure of the sentence, leaving behind the
following
"The Council on Economic Priorities began publishing Shopping for a Better World, with the simple thesis of
consumers....."
Consumers aren't a thesis. Hence, incorrect
A having + VERBED modifier cannot serve to modify only a noun.
Like a COMMA + VERBing modifier, a having + VERBed modifier must serve to modify an ENTIRE CLAUSE.
Both types of modifiers serve the same purpose: to express an action directly LINKED to the main verb.
Two key differences:
1: A COMMA + VERBing modifier serves to express an action happening AT THE SAME TIME as the main verb.
A having + VERBed modifier serves to express an action that happened BEFORE the main verb.
2: A COMMA + VERBing modifier usually appears AFTER the main verb.
A having + VERBed modifier must appear BEFORE the main verb.
Neuroscientists, having amassed a wealth of knowledge, are now drawing solid conclusions
Here, having amassed serves to express a PAST ACTION that is directly linked to a CURRENT ACTION (now
drawing).
The following would also be correct:
Having amassed a wealth of knowledge, neuroscientists are now drawing solid conclusions.
Since a having + VERBed modifier must serve to modify an entire clause, NOUN + having + VERBed is always
WRONG
----------------------

Antipoverty initiatives have had to contend with two decades of economic shifts that have
depressed wages of workers with little or no technical skills.
A. that have depressed wage of workers with little or no technical skills
B. having depressed wages of workers with little or no technical skills
C. that have depressed the wages of workers with few or no technical skills - OA
D. in which the workers wages with few or no technical skills have been depressed
E. in that workers with few or no technical skills have wages that are depressed
You can kill option B directly

Antipoverty initiatives have had to contend with two decades of economic shifts having
depressed wages of workers with little or no technical skills

--------------------------------------------------------------------------So is the "make it" correctly used here? For example, in C (if we fix the problem of agreement), "a rise in interest rates
makes it more expensive to buy on credit". Does the "it" here rightly act as a place holder for "to buy on credit", or do
we need to specify the "rise" makes WHO more expensive to buy on credits?

You can use this construction without a subject, but only if the statement is universally true -- i.e., for everyone who
might find him-/herself in the situation.
E.g., when interest rates go up, it becomes more expensive for absolutely anyone to buy on credit, because ... well,
because interest rates went up.
In a sentence describing a consequence that's particular to a certain person/group, you have to say who that is. Here,
you're talking about consequences that pertain to a certain executive, so the sentence must be specific.
-----------------------------------------------------------------------------"In" doesn't really make sense. It just notes that something has happened at some point during that period, rather
than over the course of the entire period. But this is too subtle"and favors native speakers too much"to be the kind
of thing that GMAC would test.
--------------------------------------------------In human hearing, subtle differences in how the two ears hear a given sound help the listener
determine the qualities of that sound.
A. in how the two ears hear a given sound help the listener determine
B. in the two ears hearing a given sound help the listener in determining
C. in how a sound is heard by the two ears helps the listener determine
D. between how the two ears hear a given sound helps the listener in determining
E. between how a sound is heard by the two ears help the listener in determining
The in vs. between issue is an idiom. I say differences in X (just one thing) or differences between X and Y (two
things). This sentence has one thing: how the two ears hear a given sound.
the difference between the two twins', because the two twins are two different people (as stacey points out above).
more to the point, if you were actually differentiating between the ears themselves, you would indeed say: 'the
difference between the two ears is...'
however, in this problem, you are not talking about the difference between the two ears; you're talking about the
differences in one action - the same action - that's being performed by each of the two ears. therefore, you say 'the
difference in the way the two ears perform this function.'
more illustrations:
the differences between the two twins are displayed in stark relief when they argue with each other.
the differences in the way the two twins play the violin are displayed in stark relief when they play duets together.
-------------------------------------------------e expression as much as rather than as many as to refer to the uncountable noun population.
---------------------------------------------I got the correct ans for below 1 st question i.e. A. but I took some time to eliminate ans choice D.
I am not very clear with the correct usage of "such....that''. Could you please explain that?
Could you also provide me the reasoning to eliminate choice D ?

An array of tax incentives has led to a boom in the construction of new office buildings; so abundant has capital
been for commercial real estate that investors regularly scour the country for areas in which to build.
(A) so abundant has capital been for commercial real estate that
(B) capital has been so abundant for commercial real estate, so that
(C) the abundance of capital for commercial real estate has been such,
(D) such has the abundance of capital been for commercial real estate that
(E) such has been an abundance of capital for commercial real estate,
-------------------------------------------------------------------------------------------------------------------

2).
In the minds of many people living in England, before Australia was Australia, it was the antipodes, the
opposite pole to civilization, an obscure and unimaginable place that was considered the end of the world.
(A) before Australia was Australia, it was the antipodes
(B) before there was Australia, it was the antipodes
(C) it was the antipodes that was Australia
(D) Australia was what was the antipodes
(E) Australia was what had been known as the antipodes
---------------------------------------------------------------------------------------------------------------------------------------The number of undergraduate degrees in engineering awarded by colleges and universities in the United States
increased by more than twice from 1978 to 1985.
(A) increased by more than twice
(B) increased more than two times
(C) more than doubled
(D) was more than doubled
(E) had more than doubled.
One more question ...
I was able to eliminate B and E because of the incorrect usage of '' two times'' and ''had'' but got stuck in other ans
choices.

-----------------------------------------------------------------------------------

Before Budapest was Budapest, it was Aquincum, a Celtic settlement that would go
on to become the Roman capital of Lower Pannonia.
A) Before Budapest was Budapest, it was Aquincum
B) Before there was Budapest, it was Aquincum
C) Before it was Budapest, it was Aquincum
D) Budapest was once what was called Aquincum
E) Budapest had been known as Aquincum
"Before Budapest was Budapest" seems redundant, but it's actually OK if we're
playing on two senses of the word Budapest, as we are here. What the sentence is
really saying is something like

"Before Budapest the region was Budapest the modern city, it (Budapest the region)
was Aquincum."
This is perfectly fine - short, clear, and grammatical - so A is our answer!
Looking at our other options
B: "Before there was Budapest" ... before where was Budapest? Not clear
C: What's "it"?
D: No need for a double 'was'; we'd just say "Budapest was once called Aquincum"
E: Inappropriate past perfect, we'd just say "Budapest was known as Aquincum"
------------------------------------------The British Admiralty and the War Office met in March 1892 to consider a possible
Russian attempt to seize Constantinople and how they would have to act
militarily to deal with them.
(A) how they would have to act militarily to deal with them
(B) how to deal with them if military action would be necessary
(C) what would be necessary militarily for dealing with such an event
(D) what military action would be necessary in order to deal with such an event
(E) the necessity of what kind of military action in order to take for dealing with it
correct ans: D
||ISM:
to consider
a possible Russian attempt
what military action would be necessary in order to deal with such an event
--------------------------------------------------------------------------------------------------------------------------------------------------------------------------------------While Jackie Robinson was a Brooklyn Dodger, his courage in the face of physical
threats and verbal attacks was not unlike that of Rosa Parks, who refused to
move to the back of a bus in Montgomery, Alabama.
(A) not unlike that of Rosa Parks, who refused
(B) not unlike Rosa Parks, who refused
(C) like Rosa Parks and her refusal
(D) like that of Rosa Parks for refusing
(E) as that of Rosa Parks, who refused
Ans: A

Here is some information I found on internet on the usage of 'not unlike':


It is quite common to use such a structure in English, and it's not considered a
double negative. For example, we say things like "the new law is not unfair," "the
car was not inexpensive," or "his comments were not unrelated."
We usually do this, as I explain in class (almost everybody asks the same question
about this SC), when we want a little "wiggle room," i.e., some room for error in our
sentence. Think of an adjective as having three degrees of quality, rather than only
two. Things are not often "like" or "unlike;" they often fall in the middle--not quite
"like," but not quite "unlike," either.
Let's take the above example of a car--if I told somebody I'd just bought, say, a new
Honda Accord, some people (who can afford BMW's or Mercedes') might say that
that car is an inexpensive one. I might respond that, to me, it's "not an inexpensive
car."
In this case, I'm saying that the Accord falls into the middle area, between
expensive and cheap.
moreover, the usage of ''as'' is wrong in E.''AS'' cannot be used to compare nouns.
-------------------------------------------------------------------------------------------------Danger:
we are in danger of forgetting the past.- correct
we are in danger to forget the past.- incorrect
we have a danger of forgetting the past.- incorrect
consider:
Consider X to be Y - may be correct. look for the other errors.
Consider X Y - correct
Consider X as Y- incorrect
---------------------------------------------------------------------------------------------------------------------------------------------In metal work one advantage of adhesive-bonding over spot-welding is that the contact, and hence the bonding, is
effected continuously over a broad surface instead of a series of regularly spaced points with no bonding in between.
A) instead of
B) as opposed to
C) in contrast with
D) rather than at
E) as against being at
if you don't have "at", then you wind up with over X and Y -- implying both "over X" and "over Y".
the problem is that the resulting meaning is illogical here. think about the meaning of the sentence: you have a
contrast between an application over a continuous surface (in which "over" actually makes logical sense), on the one
hand, and a different application at a series of (individual) points, on the other hand.

the issue, then, is the meaning of each preposition -- this is not a question about idioms.
when something happens at a single location, "at" is used; it makes no sense to say that something happens "over" a
single point (or several single points -- still ultimately the same issue).

--------------------------------------------------------------------------------

native to- for non-humans


native of- for humans
the usage of ''maybe '' is incorrect in GMAT. the usage of ''may be'' is completely fine.
----------------------------------------------------------------------------------------------------------------------------------So that/ Such..... that :

So happy was I that I bought flowers for everybody in class.

So quickly did she leave that we did not even realize was gone.

So rarely does a comet appear visible to the naked eye that when one does, it is considered a major event.

There are two main ways of using idiom so...that:


1. so abundant has capital been for commercial real estate that investors regularly scour the country for areas in
which to build.
2. capital has been so abundant for commercial real estate that investors regularly scour the country for areas in
which to build.
People often get confused between the above idiom and the idiom such...that. While so...that is used for the measure
of something and goes with an adjective (so abundant, so long, so beautiful), such...that is used for nature of
something and is followed by an abstract noun (such abundance, such length, such beauty)

"such NOUN that" is a perfectly respectable construction in formal English.

Conclusion:

Don is so tall that he can reach the top shelf without any effort.
Don is such a tall boy that he can reach the top shelf without any effort.=> is it correct?

i think your example here is fine, although it's unnecessary; if the emphasis is on a quality described by an adjective
("tall" in this example), then it's more compact and stylistically better to write "so + adj + that".
you are more likely to see the second of these constructions if the quality itself is embodied by a noun rather than by
an adjective. for instance, Tom is such a hermit that he hasn't left his house in two years.
in that sentence, the quality is actually described by a noun ("hermit"), so there is no option to use the "so + adj +
that" construction.
-----------------------------------------------------------------------------------------------------------------------I called him in order to invite him.
I called him to invite him.
I called him so as to invite him.
What is the change in meaning?

very interesting.
there's not really a difference in the first two. in general, "in order to" works better in longer sentences -- sentences
in which there are so many words that "to" by itself would lead to a confusing/unreadable sentence.
"so as to" is an interesting construction. i don't think that gmac likes it very much -- i've never seen it in an officially
correct answer -- but i think the main distinction is that it's used to describe indirect intentions.
for instance, the example above isn't great because you are calling this person directly to invite him, so you should
use one of the earlier two versions.
on the other hand, consider the following: Joe bought a gym membership so as to become more attractive to women.
this is what i mean by indirect intention. obviously, the act of buying a gym membership itself is not going to make joe
instantly more attractive to women (women do not swoon over a man's gym membership card). instead, there are
plenty of implied intermediate steps between "by a gym membership" and "be more attractive"; the use of "so as
to" implies that there are such intermediate steps in the process.
once again, this is a very writerly difference. i think it's fun to try to explicate it -- i've definitely never thought about
these things consciously before -- but i can guarantee that you will not be tested on this kind of difference.
-------------------------------------------------------------------------------------------------------------------------------------------------------

Out of America's fascination with all things antique have grown a market for bygone styles of furniture and
fixtures that are bringing back the chaise lounge, the overstuffed sofa, and the claw-fotted bathtub.
A) things antique have grown a market for bygone styles of furniture and fixtures that are bringing
B) things antique has grown a market for bygone styles of furniture and fixtures that is briniging
C) things that are antiques has grown a market for bygone styles of furniture and fixtures that bring
D) antique things have grown a market for bygone styles of furniture and fixtures that are bringing
E) antique things has grown a market for bygone styles of furniture and fixtures that bring
E changes the meaning, because the market -- not the styles, the fixtures, or the furniture -- is bringing back the the
chaise lounge, etc.
-------------------------------------------------------------------------------------------------------------------------------------------

Good question - I don't want you to look at this as a preposition question, which is what "to" vs. "for" is. Instead, think
of this as infinitive vs. preposition phrase that is made up of a preposition + a gerund. In general the infinitive, or "to
enhance" in your example, is used to show intent regarding another action in the sentence.

For example, "I answered your question to help you understand to vs. for." In this case, "to help..." provides the intent
associated with "I answered" and answers why I helped you.
Rephrased to take out the intent, "I hope my answer helped with understanding this topic."
In addition to intent, the infinitive is used to show reasoning. Here's an example, "I am happy to help you understand
this issue."
Finally, let's get to your example:
Like F.A. Hayek, Sowell views the concern for social justice as nothing more than a pretext to enhance/for enhancing
government power at the expense of individual freedom.
Think about what information "to enhance" or "for enhancing" conveys. Specifically how does "to enhance" or "for
enhancing" relate to the major action? First we must identify the major action, which is "views." Does "to enhance
government power..." answer why "Sowell views the concern..." this way? No it doesn't, so we should use "pretext for
enhancing."
--------------------------------------------------------------"to have VERBed" is a type of infinitive that refers to actions in a timeframe previous to the timeframe of the sentence
itself. this is actually the only kind of infinitive that can do this, so it will take the same form regardless of the tense of
the main clause.
so, if you write
"the students seem to have cheated"
then
* the sentence itself is in the present ("seem"). so, the sentence is talking about the way things appear to a present
observer.
* according to that present observer, it seems to be the case that the students cheated at some point in the past.
if you write
"the students seemed to have cheated"
then
* the sentence itself is in the past ("seemed"). so, the sentence is talking about the way things appeared to a past
observer.
* according to that past observer, it seems to be the case that the students cheated at some point earlier in the past.
-------------------------------------------------------------------------------------------------------------construction ("appear as ADJ/DESCRIPTION") is idiomatically wrong no matter what tense it's in
--------------------------------------------------------------------------------------------------------------------------------------------The first trenches that were cut into a 500-acre site at Tell Hamoukar, Syria, have yielded strong evidence for
centrally administered complex societies in northern regions of the Middle East that were arising
simultaneously with but independently of the more celebrated city-states of southern Mesopotamia, in what is now
southern Iraq.
A. that were cut into a 500-acre site at Tell Hamoukar, Syria, have yielded strong evidence for centrally administered
complex societies in northern regions of the Middle East that were arising simultaneously with but
B. that were cut into a 500-acre site at Tell Hamoukar, Syria, yields strong evidence that centrally administered
complex societies in northern regions of the Middle East were arising simultaneously with but also
C. having been cut into a 500-acre site at Tell Hamoukar, Syria, have yielded strong evidence that centrally
administered complex societies in northern regions of the Middle East were arising simultaneously but
D. cut into a 500-acre site at Tell Hamoukar, Syria, yields strong evidence of centrally administered complex societies

in northern regions of the Middle East arising simultaneously but also


E. cut into a 500-acre site at Tell Hamoukar, Syria, have yielded strong evidence that centrally administered complex
societies in northern regions of the Middle East arose simultaneously with but

* "were arising" -- this tense doesn't make sense. this is an isolated historical event, so it should be expressed in the
simple past ("arose").
this tense (i don't know what it's called) is used to describe something that was going on when something else
happened. e.g., I was sleeping when the phone rang.
can you tell me what's wrong with "having been cut" in option C? why can't we use having+done after a noun.?
please help! thanks in advance!

well, that modifier is equivalent to saying have been cut... (if it's attached to something in the present or future tense),
or had been cut... (if it's attached to something in the past tense).
e.g.
Students having finished Math 101 and 102 are eligible to take Math 201.
--> this is like saying "students who have finished..."; this tense actually makes sense here.
the problem is that this doesn't translate into the tense of the sentence in this problem, because the
trenches were cut (in the past). "have been cut" is not a sensible tense here, so the modifier "having been cut..."
doesn't work either.
-in general, you're going to find that modifiers with "having..." are somewhat rare, because they will usually (not
always!) be written with "who"/"that" when possible. for instance, the sentence above -- while not incorrect as is -would most likely be written instead as Students who have finished...
------------------------------------------------------------------------------Affording strategic proximity to the Strait of Gibraltar, Morocco was also of interest to the French throughout the first
half of the twentieth century because they assumed that if they did not hold it, their grip on Algeria was always
insecure.
(A) if they did not hold it, their grip on Algeria was always insecure
(B) without it their grip on Algeria would never be secure
(C) their grip on Algeria was not ever secure if they did not hold it
(D) without that, they could never be secure about their grip on Algeria
(E) never would their grip on Algeria be secure if they did not hold it
A and C are wrong because
the sentence talk about assumption, so hypothetical pattern "if did, would do" is required. There is no "would" in A and
C. wrong.

----------------------------------------------------------------------------------------

Confidence in X to do Y" just isn't a thing in the English language at all (= it's
"unidiomatic")
------------------------------------------------------In 1713, Alexander Pope began his translation of the Illiad, a work that,
taking him seven years until completion, and that literary critic Samuel
Johnson, Popes contemporary, pronounced the greatest translation in any
language.
A. his translation of the Illiad, a work that, taking him seven years until completion,

and that literary critic Samuel Johnson, Popes contemporary, pronounced


B. his translation of the Illiad, a work that took him seven years to complete and
that literary critic Samuel Johnson, Popes contemporary, pronounced
C. his translation of the Illiad, a work that had taken seven years to complete and
that literary critic Samuel Johnson, Popes contemporary, pronounced it is
D. translating the Illiad, a work that took seven years until completion and that
literary critic Samuel Johnson, Popes contemporary, pronounced it as
E. translating the Illiad, a work that had taken seven years to complete and literary
critic Samuel Johnson, Popes contemporary, pronounced it
The word "that" is not just repeated for clarity; the repetition is actually necessary.
The first "that" is the subject of "took him 7 years...", while the second "that" is
the object of "Samuel Johnson pronounced...". So, they are essentially different
types of words, even though they are both identical in appearance ("that").
-----------------------------------------------Here is the song that you're going to sing. (correct)
Here is the song that you're going to sing it. (incorrect)
------------------------------------This:
also, here's some "extra credit" knowledge:
there ARE constructions that can stand for the abstract information in an entire
clause (unlike pronouns, which are limited to standing for actual nouns). chief
among these are the COMMA + NOUN modifiers.
however, the presence of "because" at the beginning of choice (d) would preclude
the use of those modifiers as well.
here's an example:
studies have shown that X is 60 percent of Y, a finding that has shocked most
observers.
studies have shown that X is 60 percent of Y; this finding has shocked most
observers.
--> both correct. note that "a finding", following the comma, or "this finding",
standing alone after the semicolon, stands for the entirety of the clause that comes
before it; you couldn't use "which" here, because "which" would automatically refer
to Y.
because studies have shown that X is 60 percent of Y, this finding has shocked most
observers.
--> incorrect. the presence of "because" at the beginning of the first clause means
you can't use "this finding" anymore.
i don't have any idea what the actual rule is here, but i do know with 100% certainty
that these constructions are allowed and disallowed respectively.
------------------------------------------------------------------------------------------------

if u come across the word'' expectation''. look for the verb whether it is in correct
form.
--------------------------------------------------------------------------------------------By merging its two publishing divisions, the company will increase their share of the
country's $21 billion book market from 6 percent to 10 percent, a market
ranging from obscure textbooks to mass-market paperbacks.
A. their share of the country's $21 billion book market from 6 percent to 10 percent,
a market ranging
B. from 6 percent to 10 percent its share of the $21 billion book market in the
country, which ranges
C. to 10 percent from 6 percent in their share of the $21 billion book market in the
country, a market ranging
D. in its share, from 6 percent to 10 percent, of the $21 billion book market in the
country, which ranges
E. to 10 percent from 6 percent its share of the country's $21 billion book market,
which ranges
In other words, if you see objectively better placement"here, a choice that places
"market" directly before "which ranges...""then go ahead and eliminate the choices
with inferior placement.
--------------------------------------------------------------------"with" occupies a rather special place in the hearts of gmac's problem writers. in
other words, "with" is NOT used like other prepositions, and so, accordingly, there
are some unique points to absorb about its use.
the clearest example of this special usage is in the non-underlined part of og12
problem 29:
with individual bulls and cows receiving awards, fetching unprecedented prices, and
exciting enormous interest
this usage directly violates the principles for the use of other prepositions -specifically, "with + noun + VERBing" is allowed even though the VERBing, rather
than the noun, is the intended object of "with" . (i.e., in the sentence above, prizestock breeding was not "with bulls and cows" -- it was specifically with prizes
awarded to these animals.)
this is veeeerrry interesting, since identical constructions with other prepositions are
definitely incorrect. for instance, i've never heard of people biting dogs is incorrect,
because the intended object of the preposition is the action (the biting) rather than

the noun ("people"); instead, one could write i've never heard that people have
bitten dogs.
a similar usage can be found in og12 #23, with a past participle rather than an ING
form ("with its weight concentrated..." )
so, it is confirmed: "with" has its own special set of rules, independent of (and
contradictory to) the rules for other pronouns.
caveat lector!
-------------------------------"proof for" is unidiomatic; the correct idiom is "proof of"
----------------------------------------------------------------------------------------------------------------usage of other, another, each other & one another :
"the other" is used when there are only two things.
i chose one of the two dinner options; my date chose the other.
"another" is used for another of an indefinite number of things.
usually, i'll order one entree and my date will order another, so that we can each try
some of both.
"each other" refers to interaction between exactly two things or people.
my date and i usually sit next to each other, rather than across from each other.
"one another" also refers to interaction, but there may be more than two
things/people.
the holiday season offers many occasions for people who have not seen one
another in many months to reunite.
----------------------------------------------------------------------------------------------among other things, notice that all of the following deer-like animals are unchanged
in the plural: deer, antelope, elk, moose.
------------------------------------------------------------------------------------------------Researchers agreed that the study of new treatments for heart attack patients was
extremely important but more research was needed to determine that
balloon angioplasty
preceded with ultrasound was or was not any better for heart attack
patients than the
balloon procedure by itself.

A. more research was needed to determine that balloon angioplasty preceded with
ultrasound was or was not any better for heart attack patients than
B. more research was needed for determining whether or not balloon angioplasty
preceded by ultrasound is any better for heart attack patients than is
C. that more research was needed to determine whether balloon angioplasty
preceded
by ultrasound is any better for heart attack patients than
D. that more research was needed to determine that balloon angioplasty preceded
with ultrasound was any better for heart attack patients than
E. that more research was needed for determining that balloon angioplasty
preceded
by ultrasound is or is not any better for heart attack patients than is
The use of expression was or was not is very wordy. In this context,
whether should be used.
"determine that" is an unacceptable shift in meaning.
from the "was or was not" in the original statement -- even though that's a
redundant construction -- it's easy to figure out the intended meaning: there's no
bias toward "yes" or "no" on this particular issue.
if you replace "determine whether X is true" with "determine that X is true", then
you are now supposing that X is true, and just quibbling about how much data is
necessary to establish that for sure.
-------------------------------------------------------------------------------------A recent review of pay scales indicates that CEOs now earn an average of 419
times more pay than blue-collar workers, compared to a ratio of 42
times in 1980.
A. that CEOs now earn an average of 419 times more pay than blue-collar workers,
compared to a ratio of 42 times
B. that, on average, CEOs now earn 419 times the pay of blue-collar workers, a
ratio that compares to 42 times
C. that, on average, CEOs now earn 419 times the pay of blue-collar workers, as
compared to 42 times their pay, the ratio
D. CEOs who now earn on average 419 times more pay than blue-collar workers, as
compared to 42 times their pay, the ratio
E. CEOs now earning an average of 419 times the pay of blue-collar workers,
compared to the ratio of 42 times
"that compares to...". i say this is unidiomatic; you want "(as) compared to/with".
is wrong.
choice b is badly worded: 'compares to 42 times in 1980' seems to say that, on
forty-two different occasions in 1980, the ceo:blue-collar ratio reached 419:1. this is
not what we are trying to say.

more generally, when speaking about ratios as is done here, you cannot leave
'times' hanging like this. sometimes you can use pronouns - the height of the sears
tower is more than four times that of the statue of liberty - but you can't use empty
space.
choice c exhibits proper usage of 'times' followed by their pay. it also uses the ratio,
a correct identification of exactly what is being described.
---------------------------------------

1) The Federal investigators at Stapleton Industries have failed to find any evidence
that has suggested that the unusually large contributions to its accounts
are derived from government kickbacks, nor its officers guilty of improper
relations with industry regulators.
(A) that has suggested that the unusually large contributions to its accounts are
derived from government kickbacks, nor its officers guilty
(B) that suggests that the unusually large contributions to its accounts are derived
from government kickbacks, or its officers guilty
(C) suggesting that the unusually large contributions to its accounts had been
derived from government kickbacks, with its officers guilty
(D) to suggest that the unusually large contributions to its accounts are derived
from government kickbacks or that its officers are guilty
(E) to suggest that the unusually large contributions to its accounts had been
derived from government kickbacks, nor were its officers guilty
double negative. The verb fail in the main clause has a negative
meaning. The nor in (E) is therefore a double negative that changes the
meaning of the sentence.

3) Since the beginning of the year, the community medical clinic verified
rigorously the potency of each flu vaccine sample, lest any of its patients
are vulnerable to the flu.

(A) verified rigorously the potency of each flu vaccine sample, lest any of its
patients are
(B) rigorously verified the potency of each flu vaccine sample, lest any of its
patients would be
(C) is rigorously verifying the potency of each flu vaccine sample, lest any of its
patients not be
(D) rigorously has verified the potency of each flu vaccine sample, lest any of its
patients are
(E) has verified rigorously the potency of each flu vaccine sample, lest any of its
patients be
[adverb] + [verb]. Throughout the answer choices, the adverb
rigorously appears both before and after the verb. Which is correct?
Either is. This is a false split. They look different, but both orders are
perfectly fine.
the lest clause. A lest clause demands the subjunctive. Choices (A) & (C) use
the ordinary indicative after lest, so they are wrong. The word lest implies a
negative, so (C) has a double negative that changes the meaning of the sentence.
Choice (B) has a curious hypothetical phrasing, would be vulnerable to the flu:
this is not appropriate to this context. Only choice (E) has the correct lest clause.
5) Modern planetary science has a detailed understanding of the conditions
necessary for a planet to develop life, but scientists are still unable to determine
whether a given specific planet meeting all these conditions does develop life
(A) does develop
(B) is developing
(C) might have developed
(D) has developed
(E) could be developing
Split #1: both progressive verbs are entirely inappropriate. We dont need to
know whether life on some other planet is in the process of developing at this exact
instant. Thats not the concern of the sentence. Both (B) & (E) are wrong.

Split #2: according to the scientists, the conditions themselves already guarantee
that live could develop. If a planet meets these conditions, theres nothing
hypothetical in question we know life could develop. We want to know whether
life actually has developed. Both hypothetical answers, (C) & (E), are wrong.

Split #3: The development of life is not necessarily in the present moment, so the
present tense in (A) is incorrect. This development could have happened in the
past, up to and including the present: for this, we need the present perfect tense.
Only (D)has this.
(D) is the only possible answer.

Always put your best into everything you do


debate about"is awkward and not idiomatic
-----------------------------------------------------------------------------------------------------------------------------------------------------------each other/one another: each other is use for 2 things; one another for many.
everyday/ every day: the former is an adjective; later means daily.
--------------------------------------------------------------------------------Laboratory rats and mice live up to 40 percent longer than usual when fed a diet of at least 30 percent fewer
calories than that which they would normally eat, but that otherwise contains all necessary vitamins and
nutrients.
A. of at least 30 percent fewer calories than that which they would normally eat, but that otherwise
B. with at least 30 percent fewer calories than what they would normally eat though otherwise it
C. that has at least 30 percent fewer of the calories that they would normally eat, but otherwise it
D. that has at least 30 percent fewer calories than they would normally eat but that otherwise
E. that has at least 30 percent fewer calories than that which they would normally eat, though that

--------------------------------------------------------Adj + Adj + Noun

Vs

Adv + Adj + Noun Construction

Carbon-14 dating reveals that the megalithic monuments in Brittany are nearly 2,000 years as old as any of their
supposed Mediterranean predecessors.
(A) as old as any of their supposed
(B) older than any of their supposed
(C) as old as their supposed
(D) older than any of their supposedly
(E) as old as their supposedly
The monuments in Brittany are nearly 2,000 years older than those that had previously been dated. Thus, the issue in
not whether these monuments are Mediterranean. They are. Rather, the issue is whether the previously dated
monuments are actually predecessors. Therefore, the adjective supposed in answer choice B correctly describes the
noun predecessors, whereas in choice D the the adverb supposedly incorrectly describes Mediterranean.
Reporting that one of its many problems had been the recent extended sales slump in womens apparel, the
seven-store retailer said it would start a three-month liquidation sale in all of its stores.

(A) its many problems had been the recent


(B) its many problems has been the recently
(C) its many problems is the recently
(D) their many problems is the recent
(E) their many problems had been the recent
While some academicians believe that business ethics should be integrated into every business course, others say
that students will take ethics seriously only if it would be taught as a separately required course.
(A) only if it would be taught as a separately required course
(B) only if it is taught as a separate, required course
(C) if it is taught only as a course required separately
(D) if it was taught only as a separate and required course
(E) if it would only be taught as a required course, separately
Wrong: James Joyce is Max's SUPPOSEDLY Irish ancestor.
Right: James Joyce is Max's SUPPOSED Irish ancestor.
----------------------------------------------------------------------------------------------------Wrong: Max's grandmother is his SUPPOSED Irish ancestor.
Right: Max's grandmother is his SUPPOSEDLY Irish ancestor
See the below explanation from Ron:
it's a meaning issue -- "supposed(ly)" has to modify whatever is actually in doubt.
in the first group of sentences, the issue is whether james joyce is actually max's ancestor at all. (james joyce was
definitely irish; that's not an issue.) therefore, you want "supposed ... ancestor".
in the second group of sentences, the issue is whether max's grandmother is irish. (she is his grandmother, so she is
clearly his ancestor.) therefore, you want "supposedly irish".
------------------------------------------------------------------------------------------------------------------The golden crab of the Gulf of Mexico has not been fished commercially in great numbers, primarily on account of
living at great depths-- 2,500 to 3,000 feet down.
(A) on account of living
(B) on account of their living
(C) because it lives
(D) because of living
(E) being they live

If you want, you can just remember that "on account of ___ing" can be eliminated. I can't think of any context in which
that structure would work, so you should be able to rule it out safely.
"On account of" is followed by a noun, and is usually restricted to changes in abstract things (prices, emotions, etc.)
e.g., It's common for stock prices to fluctuate wildly on account of announcements made at press conferences.
C- CORRECT

leaching, the recovery of copper from the drainage water of mines, as a method of the extraction of minerals, it
was well established as early as the eighteenth century, but until about 25 years ago miners did not realize that
bacteria taken an active part in the process.
(A) as a method of the extraction of minerals, it was well established

(B) as a method of the extraction of minerals well established


(C) was a well-established method of mineral extraction
(D) was a well-established method of extracting mineral that was
(E) had been a method of mineral extraction, well established
E is incorrect as ''well established'' is set off by ''comma'' while it is an essential modifier.
---------------------------------------------------------------------------------Rivaling the pyramids of Egypt or even the ancient cities of the Maya as an achievement, the army of terra-cotta
warriors created to protect Qin Shi Huang, Chinas first emperor, in his afterlife is more than 2,000 years old and took
700,000 artisans more than 36 years to complete them `
A. took 700,000 artisans more than 36 years to complete them
B. took 700,000 artisans more than 36 years to complete it
C. took 700,000 artisans more than 36 years to complete
D. 700,000 artisans took more than 36 years to complete
E. to complete them too 700,000 artisans more than 36 years
since you have a parallel structure with "...and...", one way you can deduce the structure is to (temporarily) eliminate
the other half of that parallel structure.
in other words, since "is more than 2000 years old and..." is not underlined, it doesn't contribute to the structure at all,
so you can ignore it. (this is the same way in which you can, for instance, detect pronoun case more easily: for
instance, "john and i went to the theater" --> "i went to the theater".)
so, this is all you have to look at:
the army ... took the artisans more than 36 years to complete.
this is sufficient on its own, without a pronoun at the end, in much the same way as this: this test is hard to finish (not
"hard to finish it").
------------------------------------------------------------------------------------------------------------------------------------------------------------------------------

Many financial experts believe that policy makers at the Federal Reserve, now viewing the economy as balanced
between moderate growth and low inflation, are almost certain to leave interest rates unchanged for the foreseeable
future.
the reasoning doesn't apply to this one; those rules are only for comma+ ing modifiers THAT FOLLOW CLAUSES.
you can ignore "Many financial experts believe that" -- that's just a "warm up" that doesn't affect the structure or
grammar of the following clause. (test this statement by sticking it in front of any sentence at all).
therefore, that's just a nonessential modifier modifying the noun "policy makers at the FR". there's no clause to
modify, so the rules about modifying clauses don't apply.
-----------------------------------------------------------------------------------------------------------------------------------------

Soaring television costs accounted for more than half the spending in the presidential campaign of 1992, a greater
proportion than it was in any previous election.
A. a greater proportion than it was
B. a greater proportion than
C. a greater proportion than they have been
D. which is greater than was so
E. which is greater than it has been
there's ellipsis here. the understood comparison is a repeated instance of "proportion".
in contexts in which you would repeat a noun, you don't have to include the repeated noun; you may merely imply it.
this is known as ellipsis
A:

two problems here.


first, you can't have exactly the same noun on both sides of the comparison, so "it" is barred from standing for
whatever is in the other half of the comparison.
second, "it" must stand for a noun with all attached adjectives and essential modifiers -- you can't just cherry-pick the
noun, leave the modifiers behind, and assume that "it" stands for that noun by itself
----------------------------------------------------------------------The ancient Anasazi harvested such native desert vegetation as the purple-flowered bee plant, what they now
commonly call wild spinach in northern Arizona and other parts of the southwestern United States.

A. what they now commonly call


B. a plant that they now commonly call
C. now commonly called
D. and is now commonly called
E. which it is now commonly called
Brian:
Great question here...I've always enjoyed teaching this one.
For those who liked E, keep in mind that "which" is a modifier that takes the place of
"plant" - essentially "which" is used as a pronoun, so the word "it" is
redundant...we're just piling on at that point. You wouldn't say "For Christmas I got a
new bike, which it is red". You'd just say "I got a new bike, which is red..."
For A and B, I think it's important to note that "they" does properly replace
"Anasazi" as a pronoun...there's nothing wrong with the pronoun itself. What IS
wrong here is the logic. We're talking about the ANCIENT Anasazi...so it's illogical
that they would "NOW" commonly call a plant something different. Ancient means
that there isn't anything they're doing "now". A and B don't really have a pronoun
error, but they're both guilty of a vicious logical error.
C is correct - it properly modifies "plant" with "now commonly called", and therefore
is correct.
D, just for completeness' sake, illogically makes it sounds like "Anasazi" is now
commonly called "Wild spinach" - the word "and" links the two verbs "harvested"
and "is", binding them both to the same subject, Anasazi.
------------------------------------------------------------

That twenty-one ceramic dog figurines were discovered during the excavating of a 1,000-year-old Hohokam
village in Tempe, Arizona, has nearly doubled the number of these artifacts known to exist.
A. That twenty-one ceramic dog figurines were discovered during the excavating
B. Twenty-one ceramic dog figurines discovered at the excavation
C. Discovering twenty-one ceramic dog figurines at the excavating
D. Ceramic dog figurines, twenty-one of which were discovered during excavating
E. The discovery of twenty-one ceramic dog figurines during the excavation

the weirdness of the "that..." construction in (a) isn't grammatical. it's perfectly fine to use "that" + clause as the
subject of a sentence, provided that you are trying to refer to some fact as an abstraction, rather than to an actual
action.
for instance,
that you arrived on time today was a surprise to everyone.
--> this is a correct sentence. in other words, you are normally late, and so the fact that you arrived on time was a
surprise.
your arrival on time today was a surprise --> this isn't as good, because your arrival wasn't a surprise. the surprise
was the fact that you actually showed up on time.
in this particular problem, it's the opposite: the actual discovery of the figurines doubled the number of known
artifacts. (the abstract fact that these things were discovered didn't double the number.)
this is a rather subtle difference, so it may be better to rely on the distinctions that i made in the post linked above

well ... it's a subtle difference; when construction like that are used as gerunds in this kind of sentence, they are
normally preceded by "the".
however, the gmat does not explicitly test differences in the use of articles (a, an, the), so there will be other means of
elimination.
in this case, notice that "passing" is contrasted with the dedicated noun form "passage".
if you see a split between
* an -ING form that's used as a noun, and
* a dedicated noun form of the same action,
then ALWAYS pick the dedicated noun form.
Comma issue:
that's just the convention followed in english when you have city + state (or city + country, or state + country, or
neighborhood + city, or...) in the middle of a sentence.
e.g.,
Sam left his home in Red Hook, Brooklyn, and moved to Houston, Texas, soon after the big storm hit New York.

--------------------------------------------

In the mid-1970s, since birds were overcome by pollution, and routinely falling from the sky above Los
Angeles freeways, this prompted officials in California to devise a plan that reduced automobile emissions.
A. since birds were overcome by pollution, and routinely falling from the sky above Los Angeles freeways, this
prompted officials in California to devise a plan that reduced
B. since birds that had been overcome by pollution were routinely falling from the sky above Los Angeles freeways, it
prompted officials in California to devise a plan that would reduce
C. birds had been overcome by pollution and routinely fell from the sky above Los Angeles freeways, prompting
officials in California to devise a plan that reduced
D. birds overcome by pollution routinely fell from the sky above Los Angeles freeways, prompting officials in California
to devise a plan to reduce
E. birds overcome by pollution and routinely falling from the sky above Los Angeles freeways were prompting officials
in California to devise a plan to reduce

* "Since" + "prompted" = redundant.


* The tense of "reduced" doesn't make sense, because the reduction hadn't happened yet.

Ques:
I was reading a post from you. In 2009, you concluded, "it MODIFIES THE ENTIRE ACTION of the preceding clause,
and it APPLIES TO THE SUBJECT of that clause". I agree with the first half of the sentence, but I do not think the
second half (and it applies...that clause) is accurate. In this problem, birds themselves cannot perform the action of
"prompting". If birds do the action of "prompting", it will be nonsense. As you said, birds fell from the sky could not
prompt officials, only the entire preceding clause could prompt officials.
Therefore, I think when "comma+ing" demonstrates a "direct and immediate consequence of the main action" the
DOer of the "ing" can be the entire preceding clause.
Ans:
Roman, you're exactly correct. In fact, you're more than just correctthis is the main reason why the "comma +
__ing" construction exists.
Note that I hedged a bit with the language. I didn't say that the subject PERFORMS the action, nor did I say that the
subject is DIRECTLY responsible.
If that were true, a normal subject+verb pattern would probably be a better way to write the sentence.
Instead, the subject's action is responsibleusually indirectlyfor the consequence described in the "__ing"
part.
The "comma + __ing" modifier exists to make this indirect relationship clear (as opposed to subject+verb
constructions, which implydirect action/responsibility)
-------------------------------------------------

While digging in the Egyptian desert, huge fossil bones have been found by paleontologists,which appears to
have been the second most massive dinosaur that ever lived.
A. huge fossil bones have been found by paleontologists, which appears to have been
B. huge fossil bones have been found by paleontologists, which appear to be from
C. it appears that paleontologists have found huge fossil bones that are from
D. paleontologists have found huge fossil bones from what appears to be
E. paleontologists have found huge fossil bones, which are from what appear to be

"What" doesn't stand for another noun in the sentence.


In fact, the whole purpose of "what", in this kind of usage, is to refer to something that is not specifically named in the
sentence.
E.g.,
I don't know what you bought at the store.
Please tell me what you did last night.

Basically, this thing comes in two varieties.


1/
"what + subject + verb" (in which "what" is the object)
e.g.,
Show me what you bought at the store.
2/
"what + verb" (in which "what" is the subject)
e.g.,
I don't know what makes that noise in the furnace closet.

This one ("what appears to be...") is an example of #2.


-------------------------------------------------------------

Sulfur dioxide, a major contributor to acid rain, is an especially serious pollutant because it diminishes the
respiratory systems ability to deal with all other pollutants.
A. an especially serious pollutant because it diminishes the respiratory systems ability to deal
B. an especially serious pollutant because of diminishing the respiratory systems capability of dealing
C. an especially serious pollutant because it diminishes the capability of the respiratory system in dealing
D. a specially serious pollutant because it diminishes the capability of the respiratory system to deal
E. a specially serious pollutant because of diminishing the respiratory systems ability to deal
this problem is a pure test of word choice / idiomatic expression. that's a situation that i don't see often on official
problems; it's almost as though the gmat is taking cheap potshots at the (great many) non-native speakers of english
who take the test.
* "specially", which means "in a special way", is incorrect here. (example of correct use: "the specially crafted
exclusive edition of this car costs more than the standard edition") you want "especially", which means "in particular"
or "more so than all the others". therefore D and E are gone.
* "capability of ____ing" and "capability in ____ing" are unidiomatic, so B and C are gone.
those two are enough, but note also:
* "because of ____ing", where ____ing is a noun, is unidiomatic, so B and E are gone.
NOTE: be careful with this elimination. if ____ing is an adjective, not a participle, then "because of ____ing NOUN" is
a perfectly acceptable structure, as in "because of diminishing returns, i don't get as much interest from my bank
account anymore"
----------------------------------------------------------------------------------------------------------------------------------------------

According to a survey of graduating medical students conducted by the Association of American Medical Colleges,
minority graduates are nearly four times more likely than are other graduates in planning to practice in
socioeconomically deprived areas.
(A) minority graduates are nearly four times more likely than are other graduates in planning to practice
(B) minority graduates are nearly four times more likely than other graduates who plan on practicing
(C) minority graduates are nearly four times as likely as other graduates to plan on practicing
(D) it is nearly four times more likely that minority graduates rather than other graduates will plan to practice
(E) it is nearly four times as likely for minority graduates than other graduates to plan to practice
no, "plan to" is fine.
but you missed the idiom that actually _is_ incorrect in this sentence: "likely ... in planning" (incorrect, in (a)) vs. "likely
... to plan" (correct, in (b)).
"likely" must be used with an infinitive. in their usual dastardly way, the gmat writers have camouflaged this poor
idiomatic usage behind not only lots of noise (the words between "likely" and "in"), but also a second, correct idiom
(the one you singled out).
that's tough.
---------------------------------------------------------------------------------------------------

"in the last year" = within the last 12 months. this will almost exclusively be used with "has VERBed"
------------------------------------------------------------------------------------When drive-ins were at the height of their popularity in the late 1950s, some 4,000 existed in the United States, but
today there are less than one-quarter that many
A. there are less than one-quarter that many

B. there are fewer than one-quarter as many


C. there are fewer than one-quarter of that amount
D. the number is less than one-quarter the amount
E. it is less than one-quarter of that amount

in general, you won't need the second half of that construction if you have already mentioned the data elsewhere in
the sentence.
the earlier part of this sentence already mentions the # of drive-ins in the u.s. in the 1950's, so it is not stated again.
since it is not stated as part of an actual parallel construction, you don't need the second "as".
-examples:
there were once 20 shirts on this shelf, but, now, barely half as many are left. --> i already mentioned the data (i.e.,
20 shirts) earlier in the sentence, so it's not mentioned again.
there are barely half as many shirts on this shelf [i]as there were last week.[/i] --> i didn't mention this comparison
point earlier, so i'm mentioning it now.
-------------------------

Minivans carry as many as seven passengers and, compared with most sport utility vehicles, cost less,
get better gas mileage, allow passengers to get in and out more easily, and have a smoother ride.
A. Minivans carry as many as seven passengers and, compared with most sport utility vehicles, cost less,
B. Minivans, which carry as many as seven passengers, compared with most sport utility vehicles, they cost less,
C. Minivans carry as many as seven passengers, in comparison with most sport utility vehicles, and have a lower
cost, they
D. Minivans, carrying as many as seven passengers, compared with most sport utility vehicles, cost less,
E. Minivans, which carry as many as seven passengers, compared with most sport utility vehicles the cost is lower,
and they
my first complaint about (d) is that it's noticeably awkward, but you'd have to be a native speaker of english to pick up
on that.
however, there's a problem of semantics (meaning) with (d). the modifier "...carrying..." seems to signify that the latter
traits are only true of minivans when those minivans are actually carrying the requisite number of passengers. if they
aren't, then all of a sudden those things aren't true anymore.
choice (a) correctly uses the conjunction "and", which implies that both things are true but don't necessarily have to
do with each other (i.e., the # of passengers is independent of the other claims).
1) Yes, D is awkward because of this. The modifier sounds as though it could be describing "passengers."
2) Yes, the "and" helps to clarify. While the general "rule" is that noun modifiers have to touch the nouns they modify,
this is more of an art than a science. There are a few exceptions, and this is one. The other main exception is when
you have multiple modifiers for the same noun: you cannot put them all next to the same noun, so you just line them
up instead. For example, we can say: "a way of painting that became popular in the 1950's." It's not the "painting" that
became popular, but rather the "way." The word "way" doesn't make much sense on its own, however, so we have to
say "of painting" first before we modify it.
3) You can't place the comparison at the front, because the "carry as many as seven passengers" is NOT part of the
comparison. This is just a fact about minivans. In fact, this is why we've broken up the noun modifier from the noun in
the correct version, so as to preserve the meaning. Remember, modifier placements issues are issues of meaning,
and again, this is more of an art than a science - albeit a very tricky art!
------------------

it actually has to do with parallelism. consider that the sentence "splits" into two parallel tracks after the word
"minivans"; then "compared..." effectively touches "minivans". let me know if this makes sense; if not i can go into
more detail..
---------------------

Minivans carry as many as seven passengers, and compared with sport utility vehicles, most cost less, get better gas
mileage, and make it easy for passengers to get in and out, and have a smoother, more car-like ride.
A. and make it easy for passengers to get in and out,
B. and allow passengers to get in and out easily,
C. and allow passengers to get in and out more easily,
D. make it easier for passengers when getting in and out,
E. allow passengers to get in and out more easily,
---------------------------------------------------------------------------------------------------------------------------------------------------The Quechuans believed that all things participated in both the material level and the mystical level of reality, and
many individual Quechuans claimed to have contact with it directly with an ichana (dream) experience.
(A) contact with it directly with
(B) direct contact with it by way of
(C) contact with the last directly through
(D) direct contact with the latter by means of- correct
(E) contact directly with the mystical level due to
-----------------------------------------------------------------------------------------------------------------------------------------------------------can 'what' be used instead of 'that' somewhere?
If one of these makes sense, the other won't.
------------------------------------------------------------------------------------------------------------------------------------------------------------

Hello Ron,
Could you please explain the legitimate use(s) of 'what' in GMAT...

This question is too general for a forum; you'll get a better (and faster!) answer by using Google.
I can think of 2 principal ways in which "what" is used:
1/ to ask questions (irrelevant on SC)
2/ to create constructions that can act as nouns. e.g.,
Her purchases shocked her boyfriend.
What she bought today shocked her boyfriend.`
------------------"little of the ice" is correct.
Despite the growing number of people who purchase plane tickets online, airline executives are convinced
that, just as one-third of bank customers still prefer human tellers to automatic teller machine, many
travelers will still use travel agents

A) growing number of people who purchase plane tickets online, airline executives are convinced that, just as one-

third of bank customers still prefer human tellers to automatic teller machine, many travelers will

B) growing number of people who purchase plane tickets online, airline executives are convinced, just as one-third of
bank customers still prefer human tellers to automatic teller machines, that many travelers would

the easy way to eliminate (b) is to know that "would" is incorrect.

"would" can be used as a past-tense form of "will" -- for instance, i know that we will win translates into the past tense
as i knew that we would win -- or to express a hypothetical situation that isn't true. neither of these is the case here;
this is a prognostication of future events, so the future tense makes sense and the conditional ("would") doesn't.

the hard way to eliminate (b) is to realize that its construction - the placement of the commas and the word "that" isn't right.
because of the placement of the commas and "that", this choice mistakenly puts "executives are convinced" in
parallel with "1/3 of customers prefer...". that makes no sense.
in (a), though, since "that" precedes the comma, the parallelism is different: "1/3 of customers prefer..." is now parallel
to "many travelers will...", as it logically should be

Growing evidence that coastal erosion occurs continuously, not in just such calamitous bursts like hurricanes,
has led scientists and planners to urge a stringent new approach to limiting development along the nation's shoreline.

A. coastal erosion occurs continuously, not in just such calamitous bursts like hurricanes, has
B. coastal erosion occurs continuously, not just in calamitous bursts such as hurricanes, has
C. coastal erosion is continuously occurring, not in just calamitous bursts like hurricanes, having
D. there is continuous coastal erosion, not just in calamitous bursts such as hurricanes, which has
E. there is continuous coastal erosion occurring, not in just such calamitous bursts like hurricanes, has

my logic is that using both "there is" and "occuring" is redundant, because "there is continuous coastal erosion" has
already expressed the meaning of "occuring" here. So we could say either "there is continuous coastal erosion" or
"coastal erosion occurs continuously", but not both of them in the same sentence.
The idea of the sentence is to contrast "continuously" with "in calamitous bursts". "Just" modifies "in calamitous
bursts"

the easiest way to detect backward constructions is not directly, but, rather, just by figuring out that the noun(s)
that precede the verb cannot be the subject.
here are four really simple examples (all are correct):
1) there is a car in the driveway.
2) there are two cars in the driveway.
here, there's nothing in front of the verb that could even be considered as a subject -- the only word in front of the
verb is "there", which is neither a noun nor a pronoun. therefore, the subject is "car"/"cars".
3) on the table was a cell phone.
4) on the table were two cell phones.[/i]
here, "on the table" is a prepositional phrase, and so is ineligible to be the subject. therefore, the subject again must
follow the verb (since nothing in front of the verb is left to be considered), and so the subject here is "cell phone(s)".
-----------------------------

you can't block off a modifier with a comma on only one side. modifiers should be blocked off either with commas on
both sides (nonessential modifiers) or with commas on neither side (essential modifiers).
--------------------------As the honeybees stinger is heavily barbed, staying where it is inserted, this results in the act of stinging
causing the bee to sustain a fatal injury.
A. As the honeybees stinger is heavily barbed, staying where it is inserted, this results in the act of stinging causing
B. As the heavily barbed stinger of the honeybee stays where it is inserted, with the result that the act of stinging
causes
C. The honeybees stinger, heavily barbed and staying where it is inserted, results in the fact that the act of stinging
causes
D. The heavily barbed stinger of the honeybee stays where it is inserted, and results in the act of stinging causing
E. The honeybees stinger is heavily barbed and stays where it is inserted, with the result that the act of stinging
causes

first off, the construction (preposition) + NOUN + VERBing is WRONG, unless the preposition refers directly to the
NOUN. (that isn't usually the case, so, if you're in doubt, you should strike choices with this sort of construction.)

for instance:
i've never heard of bees stinging dogs
WRONG. this is not an issue of whether you've heard of bees themselves; it's an issue of whether you've heard of
their stinging dogs.

...results in the act of stinging causing...


WRONG. this doesn't result in the act of stinging itself; it results in what is caused by the act of stinging.

i have a picture of my cousin playing hockey.


CORRECT. this time, the picture is actually of my cousin, so we're good.

therefore, (a) and (d) are wrong because of "...results in the act of stinging causing...".

--

the pronoun "this" in (a) doesn't refer to any particular noun. this consideration also kills (a).

-you can't say "the stinger results in...".


"results in..." can only be used when it's LITERALLY TRUE. for instance, you could say that the attempt resulted in
failure, since the attempt ITSELF ended in failure.
if you understand this literal meaning, then it goes without saying that you can't use this sort of construction for
physical objects.
TAKEAWAY:
you can only say "X results in Y" when X is an ACTION. if X is an OBJECT, you can NEVER say that X "results" in
anything.
this kills choices (c) and (d), in which "stinger" is the subject of the verb "results".

--

choice (b) misuses the connector "as".


the connector "as" connects two complete sentences BY ITSELF. if "as" is used to connect two complete sentences,
it should NOT be used in conjunction with any other connector words.

in choice (b), "as" and "with" are used together. the use of either of these prohibits the use of the other, so this choice
is wrong.
------------------------------------------------------------------------------------------------------------------------------------------------------------

A scrub jay can remember when it cached a particular piece of food in a particular place, researchers have
discovered, and tend not to bother to recover a perishable treat if stored long enough to have rotted.
(A)tend not to bother to recover a perishable treat if
(B)they tend not to bother recovering a perishable treat

(C)tending not to bother to recover a perishable treat it


(D)tends not to bother recovering a perishable treat
(E)tends not bothering to recover a perishable treat it
the lowest-hanging fruit on that choice is the verb (at the beginning), which is plural. this is inconsistent with the
subject "scrub jay", which is singular.

there's also the problem with "if [participle]" -- a modifier that, by convention, refers to the SUBJECT of the
sentence (not the proximate noun).
for instance:
Animal 1 will attack animal 2 if injected with enough of the hormones related to aggression.
--> in this sentence it is animal 1, not animal 2, that is being injected with hormones.

so, choice (a) (as well as (c) and (e)) implies that the bird itself is "stored long enough to have rotted" -- an interesting
idea, but certainly not the intended meaning of the sentence.
still, this is a very general principle that doesn't just apply to this one particular type of modifier. after a little bit of
consideration, i can give you the following more general formulation:
if you have a modifier of the form (subordinating conjunction) + (participle), then, no matter where that
modifier is placed, it should describe the subject of the clause to which it is attached.
for instance, all of the following sentences have the same meaning:
although exhausted from a long day of work, james still played football with his son.
james, although exhausted from a long day of work, still played football with his son.
james still played football with his son, although exhausted from a long day of work.
in all of these examples, the modifier is talking about james, not james's son. (the wording of the last one is a little bit
wonky, but wonky wordings are
not tested on this exam.)
on the other hand, gmac does seem to make a fairly concerted effort to avoid repetitions such as "x that y
that z", "x to y to z", etc.
i've posted on this a couple of times on the forum, although i can't seem to find the threads right now. i do remember
one specific case of a gmat prep problem in which the correct answer used "evidence to suggest that..." instead of
"evidence that suggests that...", presumably for the same reason; there were a couple of other examples as well.
needless to say, this sort of thing will not be explicitly tested; i.e., if it shows up, there will always be genuine errors in
the choices that are meant to be wrong.
----------------------------

A Swiss government panel recommended that the country sell about half its gold reserves and this raised fears of
other countries that do the same and inundate the market.
A. reserves and this raised fears of other countries that

B. reserves, which, as a result, raised fears of other countries that


C. reserves; as a result, they feared that other countries would
D. reserves, with fears raised that other countries would
E. reserves, raising fears that other countries would
Is it "The Swiss government"? That's kind of illogical...why would they make this move with its own fears that it could
cause a bigger problem. If that were the intent of the sentence you'd have to use a transition like "despite fears" to
show that the government had fears, but still decided to make that decision.

Is it "the country"? If so, that makes it part of the recommendation ("you should sell half your gold with fear!") and
that's not really logical either.

So I'd say it's a two-part reason: one, there really isn't a clear subject of "with fears raised" and two, there isn't a
logical subject for it either. The modifier "raising fears" in E assigns "raising fears" to the action, and that makes a lot
of sense. The Swiss made this decision, and the decision raised fears around the world that this could lead to a major
problem.
------------------------redundant (there's no reason to use both "probably" and "estimated"
-----------------------According to public health officials, in 1998 Massachusetts became the first state in which more babies were born to
women over the age of thirty than under it.
A. than
B. than born
C. than they were
D. than there had been
E. than had been born
B: you can't say "born under it"; this would imply babies that were born when THEY were under the age of thirty. this
is clearly not what we mean.
Correct - A
-------------------------------------------------------------------------------------------------------------------------------------------------------In an attempt to guarantee the security of its innovative water purification method, the company required each
employee to sign a confidentiality agreement prohibiting that its water purification methods be disclosed to
companies using an analogous purification process.

a. prohibiting that its water purification methods be disclosed to companies


b. prohibiting them from the disclosing of its water purification methods to any company

c. prohibiting disclosure of its water purification methods to any company


d. that would prohibit them from disclosure of its water purification methods to companies
e. that would prohibit its water purification methods to be disclosed to a company
you can prohibit PERSON from VERBing.
you can also just prohibit X, where X is a noun (such as "the disclosure...").
(e) is also unidiomatic. you can't "prohibit X to be Y".
correct- C
----------------------------------------------

In 1914 a total of 469,000 cars and trucks were produced in the United States, but in 1929 almost twice the numbers
of trucks alone came off the assembly lines
A) the numbers of trucks alone
b) that number of trucks alone
c) the number of trucks by themselves
d) as many trucks themselves
e) as many trucks by themselves
two problems with (d).

#1
the meaning of the sentence is that the number of trucks in 1929 was almost twice 469,000.
choice (b) conveys this idea accurately, with the phrase "twice that number". see, "that number" must refer to an
actual number cited in the sentence, and there is only one such number. mission accomplished.
choice (d) DOES NOT convey this idea. that choice says "twice AS MANY trucks", which means "twice as many
trucks as were produced in 1914".
the problem is that the sentence doesn't tell us how many trucks were produced in 1914 - the only figure given is a
combined figure for cars and trucks - so this statement doesn't make any sense in context.
not to mention, the intended meaning (from the original flawed sentence) is clearly that of (b).

#2
"trucks themselves" doesn't make sense.
you don't use "x itself" unless you are trying to emphasize some element of the inherent nature of x (as opposed to
something associated with x, or with some part of x).
example: pet accessories are becoming more and more popular, even though pets themselves have maintained
constant popularity.
i.e., we want to emphasize that the second part of the sentence deals with pets themselves (as opposed to
associated things such as pet accessories).
"trucks alone", though, makes perfect sense.

"trucks BY themselves" is getting closer to the intended idea, but it's still wrong (it seems to be in contrast to "trucks
sold in packages with other things")

--------------------------------------------------------------------------------------------------

"would have ___ed" is used to speculate about the consequences of things that didn't actually happen.
e.g.,
yesterday i saw a nice table. if it had fit in my car, i would have bought it and taken it home. unfortunately, though, it
was too big.
Hypothetical subjunctive:
The subjunctive mood is used to express possibility rather than actuality. So we use the subjunctive with conditions
that are contrary to fact, hypothetical, or wishful; with demands and suggestions; and with statements of necessity.
The following sentences are all in the subjunctive mood, with the subjunctive verbs in bold.
If I were president, things would be different. (Regular indicative form would be I was)
I wish Jim were able to play the trumpet. (Regular indicative form would be Jim was)
His mother insisted that he shave his beard. (Regular indicative form would be he shaves)
The instructions recommend that she clean her shirt right away. (Regular indicative form would be she cleans)
It is necessary that you be on the conference call tomorrow. (Regular indicative form would be you are)
This is basically an inverted structure for past conditional. This sentence can be written as:
If it were not for the fusion-powered heat and radiation that rush from its core, a star would collapse under its own
weight.
More examples:
1. Open hypothetical conditional
If I were to go to just one place, I would go to Vegas.
Inverted sentence:
Were I to go to just one place, I would go to Vegas.
2. Unfulfilled hypothetical conditional
If I had booked the hotel earlier, I would have got the best room.
Inverted sentence:
Had I booked the hotel earlier, I would have got the best room

GMAT Applications
The GMAT tests the subjunctive in two primary ways. The first is with hypothetical statements (or statements contrary
to fact) that involve the pairing of were and would. Here are some examples.
If I were six feet, six inches tall, I would have a shot at playing in the NBA.
(Im not six feet, six inches tall, so Im speaking contrary to fact.)
If I were to find $20 on the ground, I would buy the latest Harry Potter DVD.
(I havent found $20 on the ground. Im just speaking hypothetically.)

My brother would have better grades in math if he were a more conscientious student.
(Hes not a very conscientious student. Were speaking contrary to fact.)
Were I to find some inspiration, I would finish this article with plenty of time to spare.
(I dont yet have the inspiration. Im speaking hypothetically.)
The key is that were should always be attached to the hypothetical condition, and would should always be attached
to the consequence.

Were it not for the fusion-powered heat and radiation that rush from its core, instead its own weight would cause a
star to collapse
A. instead its own weight would cause a star to collapse
B. instead a star would have collapsed under its own weight
C. a star would have to be collapsing under its own weight
D. a star would collapse under its own weight
E. its own weight would have caused a stars collapse
* The use of both "Were it not for..." and "instead" is redundant.
-----------------------------------------------------------------------------------------------------------------------------------------------------------Socially and environmentally responsible investing is on the rise: last year in the United States, over $2 trillion was
invested in funds that screen companies according to a variety of criteria such as adherence to labor standards,
protection of the environment, and observance of human rights.

A. according to a variety of criteria such as


B. according to a variety of such criteria, as by
C. according to such a variety of criteria as by
D. in accordance with such a variety of criteria as
E. in accordance with a variety of criteria, such as by
the best answer is definitely (a).
"according to" can be used to mean "in a manner corresponding or conforming to", as in cook the rice according to
the instructions. so, despite what is written above, "according to" is not a problem here.
-the differentiating factor between (a) and (d) is the placement of the "such" modifier.
"such" should be placed AS CLOSE AS POSSIBLE to the NOUN THAT IS EMPHASIZED in context.
here are two examples to illustrate:
* i've never seen another store with such a variety of beverages as these ones.
--> the VARIETY is the point; i've never seen a store with this many different beverages.
* i've never seen another store with a variety of such beverages as these ones.
* i've never seen another store with a variety of beverages such as these ones.
--> the BEVERAGES THEMSELVES are the point; i've seen stores with a variety of beverages before, but never a
variety of these particular beverages.

in this problem, "such as" is describing the CRITERIA, not the variety. I.e., the selection process uses criteria such
as xxxxxxx; it doesn't use a variety such as (or "such a variety as" xxxxxxxxx). we're not categorizing the variety,
we're categorizing the criteria themselves.
this consideration rules out (d); so (a) is best.
if the source of the problem -- whatever it may be -- says that (d) is better than (a), then it's a dubious source.
this is definitely the most common use of according to, but, again, it's not the only one; see above.

as proof that according to can also be used in the way in which it's used in choice (a), note the following excerpt
from the reading passage on page 29 of OG12:
The idea of the brain as an information processor-a machine manipulating blips of energy according to fathomable
rules-has come to dominate neuroscience.
in fact, there is a nice clean separation here.
in almost all cases:
if "according to _____" is set off by commas, then it is citing a source or authority.
if "according to _____" is NOT set off by commas, then it means with respect to, corresponding to, organized in
terms of, etc.

in choice (a), "according to" is used 100% properly.


----------- YOU DON'T NEED TO KNOW THE STUFF BELOW THIS LINE FOR THE TEST ----------... but it may be of interest to you if you plan to write documents in english
in fact, in accordance with is *NOT* used properly in choice (d), since we are talking about
the categories of criteria -- i.e., the rules are not specified.
the proper use of in accordance with is restricted to following rules, conventions, or established patterns.
so, for instance:
* companies are screened in accordance with criteria such as protection of human rights --> INCORRECT, since no
actual rule/convention has been stated here.
* companies A and B were eliminated from contention for the grant, in accordance with the requirement that
companies must not have any human rights violations on their record--> CORRECT; "in accordance with" a stated
rule
* we dispose of all the motor oil at approved recycling sites, in accordance with federal regulations --> CORRECT; "in
accordance with" explicitly mentioned rules.
----------------------------------------------------On Earth, among the surest indications of sunspot cycles are believed to be the rate that trees grow, as seen
in the rings visible in the cross sections of their trunks.
A. On Earth, among the surest indications of sunspot cycles are believed to be the rate that trees grow
B. On Earth, among the surest indications of sunspot cycles are, it is believed, the rate of tree growth

C. On Earth, the rate at which trees grow is believed to be among the surest indications of sunspot cycles
D. Among the surest indications on Earth of sunspot cycles, believed to be the tree growth rate
E. Among the surest indications on Earth of sunspot cycles is believed to be the rate at which trees grow
c:
some things wrong with c:
* the placement of 'on earth' is just as problematic as in choice a (again, the sentence seems to be saying that
residents of other planets disagree with us).
* 'as seen in the rings...' is a modifier that must be placed next to the thing that it modifies, which is 'the rate at which
trees grow'. the way choice c is currently written, it says that indications of sunspot cycles can be seen directly in the
rings - and it also implies that sunspot cycles (instead of trees) have rings!
I believe THAT" or "It is believed TO BE":

they're both correct. they are, however, very different.


the first is in the active voice, so the subject refers to the person holding the belief.
ex:
i believe that the moon is square.
in this case, i hold the belief that the moon is square.
the second is in the passive voice, so the subject refers to the thing about which the belief is held.
ex:
the moon is believed to be square by certain people.
in this case, the mentioned beliefs are held about the moon.
------------------------------------------------------Wisconsin, Illinois, Florida, and Minnesota have begun to enforce statewide bans prohibiting landfills to accept
leaves, brush, and grass clippings.

(A) Same
(B) prohibiting that landfills accept leaves, brush, and grass clippings
(C) prohibiting landfills from accepting leaves, brush, and grass clippings
(D) that leaves, brush, and grass clippings cannot be accepted in landfills
(E) that landfills cannot accept leaves, brush, and grass clippings

To use the word 'ban' correctly, you have to say that the ban prevents (or prohibits, or proscribes, or criminalizes, or
one of the many other words carrying the meaning of 'puts off limits') something. You can't say 'a ban that X does Y'.
So that does it for the last two choices.

As an alternative construction - possibly the most common construction, actually - you can say a ban ON (NOUN).
This construction could theoretically be used here, but (1) it isn't, and (2) it would be difficult to introduce it into this

particular sentence without some seriously awkward phrasing.

A and B are done in by poor idiomatic usage. As far as A, you can't say 'X is prohibited TO (verb)', although you can
say the related 'It is prohibited to (verb)'. You have to say that X is prohibited FROM (verb)ing'. And B should look
really, really wrong to just about any native writer/speaker of English.

That leaves C
-------------------------------------------

"WOULD" AND "COULD" These words have 2 different incarnations.

Usage #1
"Would" is the past tense of "will", and "could" is the past tense of "can" .
e.g.
According to his most recent advertisement, Mookie the Bookie can predict with 100% accuracy which
teams will win next weeks games.
vis- -vis
His October 2, 1982, advertisement declared that Mookie the Bookie could predict with 100% accuracy which
teams would win the following weeks games.

Usage #2
"Would" and "could" are used to describe hypothetical situations that are not true, or are extremely
unlikely. (since these situations are hypothetical -- i.e., they never happened -- they don't really have a
timeframe.)
e.g.
If I had one million dollars, I could buy 800,000 hamburgers at the gas station.
If I had one million dollars, I would donate 800,000 hamburgers to the county food bank.
-------------------------------------------------------------------------------------

Although exhausted, Jim managed to keep himself awake for the duration of his three-hour drive home.
or
Jim, although exhausted, managed to keep himself awake for the duration of his three-hour drive home.
both correct
---------------------------

still + continues--- redundant


----------------------------

''as '' vs ''with'' when ''as'' is used in in case of reason, any answer choice that replaces ''as'' -> ''with'' will be incorrect.
---------------------------with the aim of- correct
with the aim to- incorrect
-----------------------------------------------------------------------------------------------------------------------------------------------------------Unlike transplants between identical twins, whose genetic endowment is the same , all patients receiving hearts
of other organs must take antirejection drugs for the rest of their lives.

A. Unlike transplants between identical twins, whose genetic endowment is the same
B. Besides transplants involving identical twins with the same genetic endowment
C. Unless the transplant involves identical twins who have the same genetic endowment
D. Aside from a transplant between identical twins with the same genetic endowment
E. Other than transplants between identical twins, whose genetic endowment is the same

In A and B, the phrases beginning Unlike and Besides modify patients, the subject of the main clause; thus A absurdly
states that Unlike transplants, patients must take drugs, and B that all patients except for transplants must take
drugs.
In B and D the expression identical twins with the same genetic endowment wrongly suggests that only some
identical twin pairs are genetically identical.
In E, the construction Other than transplants, all patients must take drugs illogically suggests, as in B, that some
patients are transplants.
Choice C, the best answer, solves these problems by using a clause introduces by Unless to describe the exception
to the rule and a nonrestrictive clause beginning with who to describe the characteristic attributed to all identical
twins.
Answer: C.
-----------------------------------------------------------------------------------------------

excepting a concert performance that the composer himself staged - excepting would never be correct in GMAT.
---------------------------1) sufficient + enough is REDUNDANT. redundancy is very bad.
same reason you wouldn't say "reply back", "added bonus", or "determined as a result of" (see #138 in OG11, if you
have that).
(2) make sure you know that "enough THAT" is incorrect.
the 2 idioms mentioned above are correct
--------------------------

in January 1994 an oil barge ran aground off the coast of San Juan, Puerto Rico, leaking its cargo of 750000
gallons into the ocean, while causing the pollution of the city's beaches.
A. leaking its cargo of 750000 gallons into the ocean, while causing the pollution of
B. with its cargo of 750000 gallons leaking into the ocean, and it polluted
C. and its cargo of 750000 gallons leaked into the ocean, polluting

D. while it leaked its cargo of 750000 gallons into the ocean and caused the pollution of
E. so that its cargo of 750000 gallons leaked into the ocean, and they were polluting.
not simultaneous action. one of these is the CONSEQUENCE of the other; they are not two separate events
happening at the same time.
if you say "X-ing while Y-ing", then you must be describing two separate but simultaneous actions.
ques:
Would it have been correct to say i.e. removing "while" from A ?
"leaking its cargo of 750000 gallons into the ocean, causing the pollution of"
ans:
no. you can't use two comma+ing modifiers in a row.
you could make that choice reasonable with a little further editing, i.e., "leaking its cargo of 750,000 gallons into the
ocean and thus causing..."
------------------------------On account of a law passed in 1993, making it a crime punishable by imprisonment that a United States
citizen hold gold in the form of bullion or coins, immigrants found that on arrival in the United States they had to
surrender all of the gold they had brought with them.
A. On account of a law passed in 1993, making it a crime punishable by imprisonment that a United States citizen
hold
B. With a law passed in 1933 that makes it a crime punishable by imprisonment that a United States citizen hold
C. A law passed in 1933 that made it a crime punishable by imprisonment for a United States citizen holding
D. Because of a law passed in 1933 making it a crime punishable by imprisonment for a United States citizen to hold
E. Due to a law being passed in 1933 that makes it a crime punishable by imprisonment for a United States citizen to
hold
a, b, c can all be eliminated because of improper idiomatic usage: the proper idiom is '...a crime for blah blah
blah to hold...'
'with' in choice b is also bad: it seems to imply that immigrants arrived with the law in their hands.
choice c implies that the immigrants themselves are 'a law passed in 19xx' (analogy: 'an accomplished pianist, jay
made a nice living playing at weddings' - jay is an accomplished pianist. same reading applies to this sentence,
although it's considerably longer and more difficult to parse)
choice e changes the meaning of the sentence: taken literally, it says that the immigrants found themselves in this
unenviable situation not because of the law itself, but because of the passage of the law. although somewhat
plausible, this is not the intent of the original sentence.
well, there are TWO essential modifiers in that sentence, both of which are traditionally placed after the noun. (both of
them are participial modifiers - one a present participle, one a past participle)
* passed in 1933
* making it a crime...
since we can't place both of these modifiers directly after the noun, we have to place one of them after the other.
since "passed in 1933" is the shorter of the two, we elect to place that one after the noun.**
-**this is often the "rule" that's used for the placement of two items that have the same grammatical priority - i.e., you
don't know which one to place first, since they're both things that go in the same place.
for instance:
i dedicated a song to my father

i dedicated to my father a song that recounted all the lessons he taught me in life
both of these are correct constructions; if you reverse the placement in the second one, it becomes too confusing /
difficult to read
-----------------------------------------------------------------Inuits of the Bering Sea were in isolation from contact with Europeans longer than Aleuts or Inuits of the North
Pacific and northern Alaska.
(A) in isolation from contact with Europeans longer than
(B) isolated from contact with Europeans longer than
(C) in isolation from contact with Europeans longer than were
(D) isolated from contact with Europeans longer than were
(E) in isolation and without contacts with Europeans longer than
this is (d).
* "isolated" is plainly better than "in isolation", so we can narrow the problem to (b) and (d) right away.
and if you entertained the thought of choosing (e) even for two seconds, you really need to read through a bunch of
correct answers - just read through the correct answers - to get a handle for what the correct sort of english looks like
on these things. "in isolation and without contacts" is not only laughably wordy, but also redundant (it says exactly the
same thing twice).
before we look at the actual difference between (b) and (d), a general takeaway:
* if you have 2 ANSWER CHOICES THAT DIFFER ONLY IN THE PRESENCE/ABSENCE OF A LITTLE WORD,
then CHANCES ARE THAT THE LITTLE WORD IS NECESSARY. i can't guarantee this 100%, but LOOK FOR
AN AMBIGUITY that arises if the little word is omitted.
* if the little word isn't the _sole_ difference between 2 answer choices, then try to concentrate on the other
differences between the answer choices first.
in this problem, (b) is ambiguous. it could mean one of the following 2 things: ("BI" = bering inuits, "E" = europeans,
"A" = aleuts etc.)
* BI were isolated from E longer than BI were isolated from A
* BI were isolated from E longer than A were isolated from E
we can't tell.
if you add the "were", then the meaning is narrowed down to the latter of these two possibilities.
--------------------------------------------------------------Sound can travel through water for enormous distances, prevented from dissipating its acoustic energy as a
result of boundaries in the ocean created by water layers of different temperatures and densities.
A) prevented from dissipating its acoustic energy as a result of
B) prevented from having its acoustic energy dissipated by
C) its acoustic energy prevented from dissipating by
D) its acoustic energy prevented from being dissipated as a result of
E) preventing its acoustic energy from dissipating by

"as a result of" doesn't make sense, because (as you can discern from context) the energy is actually prevented
from dissipating BY the boundaries.
"as a result of" suggests indirect causation. i.e., when you write "x happened as a result of y" , the
implication is that y didn't cause x directly, although y was in some way ultimately responsible for x.
e.g.
if 500 people actually died IN an explosion, you could write
500 people were killed by the explosion.
on the other hand, if some or all of these people didn't actually die in the explosion itself, but died from other causes

related to the explosion -- e.g., the explosion released carbon monoxide, which ultimately killed them later -- then that
sentence would be inaccurate. in this case, you could write
500 people were killed [i]as a result of the explosion[/i].
---------------------------------------------

"Influential on" is bad idiomatic usage.


-------------------------------------Initiated five centuries after Europeans arrived in the New World on Columbus Day 1992, Project SEIT pledged a
$100 million investment in the research for extraterrestrial intelligence.
(a) the same
(b). Initiated on Columbus Day 1992, five centuries after Europeans arrived in the New World, a $100 million
investment in the research for extraterrestrial intelligence was pledged by Project SEIT.
(c). Initiated on Columbus Day 1992, five centuries after Europeans arrived in the New World, Project SEIT pledged a
$100 million investment n the research for extraterrestrial intelligence.
C:
"five centuries after Europeans arrived in the New World" modifies "columbus day 1992". so, in effect, you have a
modifier within another modifier:
(Initiated on Columbus Day 1992(, five centuries after Europeans arrived in the New World,)) Project SETI pledged a
$100 million investment n the research for extraterrestrial intellingence
-------------------------------In A.D. 391, resulting from the destruction of the largest library of the ancient world at Alexandria, later
generations lost all but the Iliad and Odyssey among Greek epics, most of the poetry of Pindar and Sappho, and
dozens of plays by Aeschylus and Euripides.
(A) resulting from the destruction of the largest library of the ancient world at Alexandria,
(B) the destroying of the largest library of the ancient world at Alexandria resulted and
(C) because of the result of the destruction of the library at Alexandria, the largest of the ancient world,
(D) as a result of the destruction of the library at Alexandria, the largest of the ancient world,
(E) Alexandria's largest library of the ancient world was destroyed, and the result was
ques:
How do we know that the initial modifier 'Resulting from....' is modifying
Case 1. The subject (or noun) that follows the comma. Here, it is 'later generations'
Case 2. The entire clause that follows comma 'later generations lost all but the Iliad and Odyssey among Greek epics
and dozens of plays by Aeschylus and Euripides'
If it is Case 1, then this will be the reason to eliminate the choice. But if that is not the case, how can I eliminate
option A.
ans:
BOTH #1 AND #2 are necessary for that modifier to work.
since #1 does not apply, the modifier is incorrect.
-------------------------------------------------in your cases #2 and #3, the modifiers can't modify only the subject; there must be some sort of
relevance to the action of the verb.
in other words, if this kind of modifier accurately describes the subject but has nothing to do with the
verb/action, then it's incorrect.
e.g.
Standing almost seven feet tall, Wade is one of the best math students in the class.
--> incorrect; even though "standing almost seven feet tall" describes "Wade", there is no reasonable
connection between wade's height and his math prowess
vs.

Standing almost seven feet tall, Wade is one of the most physically imposing students in the class.
--> correct; the modifier applies to wade and also has an obvious relationship to the following clause.
-by the way, i can also think of a fourth usage of this kind of modifier: it may follow a noun (i.e., NOT a
clause), in which case it modifies that noun.
still, as before, it should also have some kind of obvious relationship to the following action.
e.g.
Wayne, frantically waving his arms, tried to attract the attention of the pilot flying overhead.
(correct example)
--> in this sentence, note that (a) the modifier "waving..." describes wayne, and also that (b) there is a
clear relationship between the action of the modifier and the action of the clause that it modifies.
-------------------------------------------------------------------------------------------------------------------------------------------------------------------------

from what i've seen, 'assist ... in' is the generally preferred form. i'm not so confident as to say that 'assist X to do y' is
just plain wrong, but 'in' is preferred
assist in- correct
assist to- incorrect
--------------------------------------------------------------------------------------------------------------------------------------both "distinguish between X and Y" and "distinguish X from Y" are legitimate idioms.
----------------------------------------------------

At an orientation meeting, the travelers were told that a visa, a landing card, and evidence of
inoculation against typhoid fever would be needed by each of them.
A. a visa, a landing card, and evidence of inoculation against typhoid fever would be needed by each of
them
B. they would need a visa, a landing visa, and evidence of their being inoculated against typhoid fever
C. they would need evidence of being inoculated against typhoid fever and a visa and landing card
D. they would each need a visa, a landing card, and evidence of inoculation against typhoid fever
E. they would need visas, landing cards, and evidence of inoculation against typhoid fever for each of
them.
SC questions on the GMAT almost never test whether articles are necessary, so you
shouldn't worry about that. Maintaining the same article is not necessary for parallelism.*
Generally speaking, articles are used to convey meaning about the number of the noun in
question.

"A" specifies one specific thing out of many: "Travelers... would each need a visa"
implies that there are lots of visas out there in the world, but each traveler only needs one. A
visa is one specific, tangible object.
"The" means the only one of something, or all of something: If we say "the
evidence," it implies all the evidence. For example, "the jury considered the evidence
against the accused" means that the jury considered every piece of evidence.
"Evidence" is not a specific object, but a term given to a range of objects that serve to
provide proof. In this sentence, travelers don't need to provide "the evidence," because it
doesn't need to be a comprehensive collection of evidence. We also can't say "an evidence",

because it's not any specific object.


No article means "some" or an undefined amount: "Travelers [need] evidence of
inoculation" just means "some evidence," so this construction is fine. The nouns are parallel,
and we don't need to match the articles - that would change the meaning.
*The only time that articles are necessary to parallelism is when a present participle is being
used as a noun (gerund). For example, in OG #119: "New theories propose that catastrophic
impacts of asteroids and comets may have caused reversals in the Earths magnetic field,
the onset of ice ages, the splitting apart continents 80 million years ago, and great volcanic
eruptions."
"The" was necessary to turn "splitting" into a noun.
---------------------------------------------------

Unlike most severance packages, which require workers to stay until the last day scheduled to collect, workers
at the automobile company are eligible for its severance package even if they find a new job before they are
terminated.
A) the last day scheduled to collect, workers at the automobile company are eligible for its severance package.
B) the last day they are scheduled to collect, workers are eligible for it at the automobile company's severance
package.
C) their last scheduled day to collect, the automobile company offers its severance package to workers.
D) their last scheduled day in order to collect, the automobile company's severance package is available to workers.
E) the last day that they are scheduled to collect, the automobile company's severance package is available to workers
if you say "the last day that they are scheduled to collect", this means that "day" must be a direct object.
this makes no sense, unless the employees are collecting days (which they clearly aren't).
-another example:
"the time that we were scheduled to meet" is incorrect. the literal reading of this (which is the only reading you care
about, by the way) is that you were scheduled to meet the time itself. (ron: "hi, time!" time: "hi, ron!")
(in case you're wondering, the correct way to say this would be "the time at which we were scheduled to meet", since
the preposition "at" must be conserved.)
----------------------------

Often major economic shifts are so gradual as to be indistinguishable at first from ordinary fluctuations in
the financial markets.
a. same
b. so gradual so that they can be indistinguishable
c. so gradual that they are unable to be distinguished
d. gradual enough not to be distinguishable
e. gradual enough so that one cannot distinguish them

if this is an official problem, then i guess the issue boils down to the fact that the incorrect answer (c) is
substantially more wordy than the correct answer (a).
i also find something a bit awkward about the use of "unable" to describe something that's not actually an
inability of the subject.
here's what i mean:
this plant is unable to perform photosynthesis.
legitimate, because this actually describes an inability of the plant itself.
this sentence:
they (= major economic shifts) are unable to be distinguished...
i don't like this, because we're not actually talking about an "inability" possessed by the economic shifts
themselves.
if we'd said something like "economic shifts are unable to destroy your equity", then i'd find that more
appealing
----------------------------------

Omission of ''that'':
Trans World Entertainment Corporation, which owns the Record Town and Saturday Matinee retail chains, announced that since sales of up to
one-fourth of its stores are poor, they will be closed.
A) that since sales of up to one-fourth of its stores are poor, they will be closed
B) it is closing up to one-fourth of its stores, which accounted for its poor sales
C) It was closing up to one fourth of its stores because of poor sales
D) to be closing, on account of poor sales, up to one-fourth of its stores
E) having poor sales, such that up to one-fourth of its stores will be closed
The default referent for a subject pronoun is the SUBJECT OF THE PRECEDING CLAUSE.
In A, they (subject pronoun) seems to refer to sales (the subject of the preceding clause), implying that SALES will be closed.
Not the intended meaning.
The intended meaning is that the unsuccessful STORES will be closed.
Eliminate A.
B: its stores, which accounted for its poor sales
Here, which seems to refer to its stores, implying that ALL of Trans World's stores were responsible for the company's poor sales.
Not the intended meaning.
The intended meaning is that only ONE-FOURTH of the stores were unsuccessful.
Eliminate B.
Generally, an infinitive modifier serves to refer to the PRECEDING SUBJECT.
D: Trans World Entertainment Corporation...announced to be closing.
Here, announced to be closing is unidiomatic.
Worse, the implication of to be closing is that Trans World Entertainment Corporation (the preceding subject) is about TO BE CLOSING.
Not the intended meaning.
The intended meaning is that UP TO ONE-FOURTH OF THE STORES will be closing.
Eliminate D.
E: announced having poor sales
Here, having seems to serve as the direct object of announced, implying that Trans World announced the ACT OF HAVING POOR SALES.
Not the intended meaning.
The intended meaning is that the company announced IT WAS CLOSING UP TO ONE-FOURTH OF ITS STORES.
Eliminate E.
The correct answer is C.
In most cases on the GMAT, a reporting verb such as announced will be followed by that.
This SC -- which is from GMATPrep -- illustrates an important exception to this rule:
When a subject is reporting information about ITSELF, the GMAT may omit that from the noun-clause serving as the direct object of the
reporting verb.
The typical construction is as follows:
SUBJECT + REPORTING VERB + it + VERB.
OA: Trans World Entertainment Corporation announced it was closing up to one fourth of its stores.
Here, Trans World is reporting information about itself.
As a result, it is permissible to omit that after the reporting verb announced.
SC48 in the OG for Verbal:
The seven-store retailer said it would start a three-month liquidation sale.

Here, the seven-store retailer is reporting information about itself.


As a result, it is permissible to omit that after the reporting verb said.
In some cases, the GMAT will INCLUDE that when the subject is reporting information about itself.
SC79 in the OG for Verbal:
The computer company has announced that it will purchase the color-printing division of a rival company.
Here, even though the computer company is reporting information about itself, the reporting verb has announced is followed by that.
Bottom line:
If a subject is reporting information about itself, a reporting verb such as announced may or may not be followed by that.
Thus, do not eliminate an answer choice solely because it includes or omits that after the reporting verb.
Instead, look for other reasons to eliminate answer choices.

------------------------Correct usage of ''due to'':


it always modifies noun.
to make sure that its usage is correct, replace it with ''caused by'' and see it is still correct.
-----------------------------THE ONLY PRONOUNS ON THE GMAT THAT DON'T HAVE TO STAND FOR NOUNS:
It + description + that + complete sentence (independent clause)
It + description + to + verb (infinitive)
It + (TO BE verb) + NOUN + that/who + verb
these "it"s DO NOT have to stand for nouns.
ALL other gmat pronouns must stand for nouns.
FIRST TYPE
It + is obvious + that Medelln will beat Nacional in Wednesdays game.
(for any paisas who might be reading this --vamox medallo!)
It + has been said + that the moon is made of green cheese.
Johnny found it + counterintuitive + that the GMAT tests exceptions to rules just as often as it tests the rules themselves.
SECOND TYPE
It + is often difficult + to distinguish between a past-tense verb and a past participle.
The rain made it + quite challenging + to drive on the freeway.
THIRD TYPE
It + was my own brother + who committed the crime.
--------------------------

Scientists have dated sharp-edged flakes of stone found in the fine-grained sediments of a dry riverbed in the Afar
region of Ethiopia to between 2.52 and 2.60 million years ago, pushing back by more than 150,000 years the earliest
date when it is known that humans made stone tools.
A. when it is known that humans made
B. at which it is known that humans had made
C. at which humans are known to have made
D. that humans are known to be making
E. of humans who were known to make
probably the easiest way to go here is to remember this as an idiomatic usage of the construction "known to". if the
action is in the present, then you use "known to VERB"; if the action is in the past, then you use "known to have
VERBed". as far as i know, these are the only two possible forms.

According to analysts, an alliance between three major personal computer companies and most of the nations largest
local telephone companies would enable customers to receive internet data over regular telephone lines with speeds
much higher than is currently possible.
A. with speeds much higher than is
B. with speeds that are much higher than are
C. at much higher speeds as are
D. at much higher speeds than that
E. at speeds much higher than are
1. In the option A, if we used "ARE" in stead of "IS", would it be correct?
No, the subject is speeds: the speeds ARE currently possible. Also, the correct idiom is that you travel not with a given
speed but at a given speed.
2. What is the exact cause for which option D is wrong?
the pronoun that is singular, but it's referring to speeds, a plural noun
------------------------------------------------------------------------------------------------------------------------------------------------------------------------------------------------------

Researchers in Germany have unearthed 400,000-year-old wooden spears from what it appears was an ancient
lakeshore hunting ground as stunning evidence of human ancestors who systematically hunted big game
much earlier than believed.

(A) it appears was an ancient lakeshore hunting ground as stunning evidence of human ancestors who
(B) it appears had been an ancient lakeshore hunting ground and is stunning evidence that human ancestors
(C) it appears to have been an ancient lakeshore hunting ground and is stunning evidence that human ancestors
(D) appears to be an ancient lakeshore hunting ground, stunning evidence that human ancestors
(E) appears that it is an ancient lakeshore hunting ground, stunning evidence of human ancestors who

The GMAT seems to be trying to trick us into thinking that "it appears" is correct, because it's a commonly used
expression on its own. "It appears to be raining outside," "it appears that they are running late," etc. We can't use both
"what" and "it" in the same expression, though.
In colloquial English expressions, we often use "it" to refer to general states of being: "it's sunny," "it was a shame that
you couldn't come," etc. In these examples, "it" doesn't replace a specific noun, but refers to a general situation.
Although these are perfectly correct constructions, the GMAT tends not to use "it" in this way. When you see "it" on
the GMAT, it should refer to a specific antecedent clearly defined in the sentence. (There may be a handful of
exceptions, but this is almost always the case).
The correct idiomatic usage of "appears" is either "something appears to be" or, more colloquially, "it appears that
something is/was..." For this reason, A, B, and E are not idiomatically correct.
---------------------------------------------------------------------------------------------------------------------------------------"to ___" > same timeframe as the larger sentence/context in which it appears
"to have ___ed" > earlier than the timeframe of the larger sentence/context
E.g.,
You appear to be injured. (It seems that you're injured right now.
---------------------------------------------

You appear to have been injured at work. (You may still be hurt; you may be OK now. The injury happened earlier.)
--------------------------------------

For the farmer who takes care to keep them cool, providing them with high energy feed and milking them regularly,
Holstein cows are producing an average of 2275 gallons of milk each year.
A. providing them with high energy feed and milking them regularly, Holstein cows are producing
B. providing them with high energy feed ,and milked regularly, the Holstein cow produces
C. provided with high energy feed, and milking them regularly, Holstein cows are producing
D. provided with high energy feed ,and milked regularly, the Holstein cow produces
E. provided with high energy feed ,and milked regularly, the Holstein cows will produce
heh. it looks like choice (a) is an "indian trap"
seriously, not a joke.
if there is a single biggest issue in the grammar of second-language english speakers who happen to hail from india or
pakistan, that issue is the drastic overuse of the "are ...ing" construction. (native speakers of english will recognize
"are producing" at once as awkward.)
another problem with part (a) is the modifier. although "providing... and milking..." is a grammatically acceptable
modifier, it doesn't make any sense in context, since these are not two things that farmers do
or AS A CONSEQUENCE OF keeping the cows cool. (if you're going to use comma + -ing, then one of these two
should hold.)
see #124 (og 12th edition) or #127 (og 11th edition), correct answer, for a sentence in which such a modifier
actually does make sense.
the official answer, (e), is tricky indeed.
you have to parse it as follows:
you KEEP THE COWS cool
you KEEP THE COWS provided with...
and you KEEP THE COWS milked regularly
ugly. very ugly. but (e) is the least ugly of the bunch, so it wins the proverbial pageant.
i would much prefer "keep them cool, provide them with..., and milk them...", but that isn't there, of course.

------------------------------------------------------------------------------------------------------------------------------------------Turning away from literary realism to write romantic stories about the peasant life and
landscape of northern Sweden, in 1909 Selma Lagerlf was the novelist who became the
first woman and was also the first Swedish writer to win the Nobel Prize for Literature.
A. Turning away from literary realism to write romantic stories about the peasant life
and landscape of northern Sweden, in 1909 Selma Lagerlf was the novelist who
became the first woman and was also the first Swedish writer to win
B. She turned away from literary realism and wrote romantic stories about the
peasant life and landscape of northern Sweden, and novelist Selma Lagerlf in
1909 became the first woman as well as the first Swedish writer that won
C. Selma Lagerlf was a novelist who turned away from literary realism to write
romantic stories about the peasant life and landscape of northern Sweden, and in
1909 she became the first woman in addition to the first Swedish writer winning
D. A novelist who turned away from literary realism to write romantic stories about
the peasant life and landscape of northern Sweden, Selma Lagerlf became in
1909 the first woman and also the first Swedish writer to win

E. As a novelist, Selma Lagerlf turned away from literary realism and wrote
romantic stories about the peasant life and landscape of northern Sweden, in 1909
becoming the first woman and also the first Swedish writer that won
"as a novelist" implies that you're going to talk about selma in some other capacity later ("as a novelist, she did X; as a
woman, she did Y"). so we don't want that.
yeah, "in 1909" is in the wrong place. i'd place it after "becoming" if i had to throw it somewhere in that modifier.
the worst thing about this choice, though, is "that". you cannot EVER use "that" to refer to people, even though we do
so all the time in spoken english.

The normative model of strategic decision-making suggests that executives examine a firms external environment
and internal conditions, and in using the set of objective criteria they derive from these analyses, can decide on a
strategy.
A. conditions, and in using the set of objective criteria they derive from these analyses, can decide
B. conditions, and they use the set of objective criteria derived from these analyses in deciding
C. conditions and, in using the set of objective criteria derived from these analyses, deciding
D. conditions and, using the set of objective criteria derived from these analyses, decide
E. conditions and, in their use of the set of objective criteria they derive from these analyses, they
decide
The problem with choice B is that, at best, it's ambiguous. At worst, it suggests an incorrect interpretation.
B has 2 possible meanings:
1/
(a) The model suggests that executives do xxxx, and (b) they do yyyy.
2/
The model suggests (a) that executives do xxxx and (b) that they do yyyy.
Either interpretation is possible. Worse, the punctuation of choice B strongly suggests the first (incorrect)
interpretation.
The correct answer, with its two parallel verbs, has no such ambiguity.
analogy:
the law stipulates that i take a break for at least 30 minutes, and i often take one for much longer.
see how this works?
the law stipulates that i take a break for at least 30 minutes
i often take a longer break (which, of course, has nothing to do with the law)
same thing with B. but, unlike the sentence above, B doesn't work. (the model is a model of decision-making, so
both things are clearly part of the model.

ques

Moreover, "use X in doing Y" seems to be not an idiom.


"use x to y" is correct.

No, "use __ in __ing" is a perfectly respectable construction, provided that it actually makes sense.
If "x" is a tool or process that's directly involved in doing "y", then the second version is better, because it conveys that
idea. However, if "x" is NOT such a tool and makes only an indirect contribution, then "use __ to __" is wrong, while
the first version can work.
E.g.,
Matthew used nothing but Pepsi bottle caps to make his 100-foot-tall sculpture of Vinminen.
(i.e., the sculpture is actually made of bottle caps)
But...
Matthew, who is afraid of heights, made extensive use of meditation techniques in making his 100-foot-tall
sculpture of Vinminen.
Here, "to make" would be wrong, because meditation plays no part in the actual construction of a sculpture. This
version, on the other hand, correctly conveys the indirect contribution of meditation, presumably to counter the
sculptor's fear of heights.

---------------------In the sixteenth century, the push for greater precision in measuring time was not, like more
recently, motivated by complicated philosophical questions about the nature of matter and
the universe, but the practical matters of navigation: sailors simply needed more highly
accurate timepieces in order to compute their longitude form the positions of the stars.
A. not, like more recently, motivated by complicated philosophical questions about the
nature of matter and the universe, but the practical matters of navigation
B.being motivated by the practical matters of navigation, instead of complicated
philosophical questions about the nature of matter and the universe, as it has been
recently
C.motivated not by complicated philosophical questions about the nature of matter and the
universe, like they were more recently, but by the practical matters of navigation
D.motivated by the practical matters of navigation, not complicated philosophical questions
about the nature of matter and the universe, which was the case more recently
E.motivated not by complicated philosophical questions about the nature of matter and the
universe, as has been the case more recently, but by the practical matters of
navigation- correct
* this is the TRADITIONAL USE OF "AS", to refer to a CLAUSE (note that there is a TENSED VERB "has been").
you should think of FORM OF "BE" + THE CASE as an idiomatically correct expression (i.e., there doesn't have to be
a subject). it counts as a CLAUSE, since you have a tensed verb.
if this expression comes at the BEGINNING of a sentence, then you use "it" as the subject (no antecedent needed): "IT
is the case that..."
this is in much the same way that you'd say "IT is impossible that...", etc.
One more point:
with PREPOSITIONAL PHRASES, you use "AS", not "like".
ex:
in the 14th century, AS in the preceding several centuries, many wars were fought between neighboring countries

-----------------------------------------------------------------------------

Unlike that of earlier works on slavery, Blassingame's innovative study relies not on the
records of White slave owners but on the records of the salves themselves, especially the 70
or so autobiographies and memoirs that have been preserved.
A. that of earlier words on slavery, Blassingame's
B. that of earlier works on slavery, Blassingame in his
C. earlier works on slavery, Blassingame in his
D. earlier works on slavery, Blassingame's
E. the earlier works on slavery, Blassingame in his
"that" when used with "of" creates a possessive pronoun that needs to refer to some
characterstic of the earlier studies, rather than the studies themselves.

--------------------------------------------------------

Correct usage of ''Like''


Like is a comparison marker that often causes confusion in terms of its usage on the GMAT. Lets understand how to use like
correctly. Well start with a few simple examples and then discuss some official questions.
The correct usage of like in a sentence ensures that the following four conditions are met:
Like shows similarity between two noun entities.
Like presents a logical comparison.
Like is followed by a noun/pronoun. (It cannot be followed by a clause)
The sentence conveys the meaning clearly. There should be no ambiguity in the meaning.
Lets have a look at a few questions. In the following sentences, try to identify:
Which of these sentences are correct/incorrect?
What are the meanings conveyed by these sentences?
Which entities are being compared?
1. The US needs a president like Lincoln.
2. Mike can play the guitar like a professional guitarist.
3. Tom needs a gym instructor like James.
4. Like my brother, I want to be an investment banker.
Now, lets discuss the above sentences one by one:
1. The US needs a president like Lincoln.
This sentence tells us that the US needs a president similar to Lincoln The comparison is between president and Lincoln. So, the compared entities are
logically parallel. The sentence conveys perfectly clear meaning without any ambiguity.
Lets take one more example:
India needs an opening batsman like Sachin Tendulkar.
Here the comparison is between opening batsman and Sachin Tendulkar. The meaning is that India needs an opening batsman who is similar to
Sachin Tendulkar: i.e. the comparison is between an opening batsman and Sachin Tendulkar. There is no ambiguity here.

2. Mike can play the guitar like a professional guitarist.


This sentence compares Mike with a professional guitarist. It conveys the meaning that Mike is as good at playing the guitar as a professional guitarist
is.

3. Tom needs a gym instructor like James.


This sentence presents two possible comparisons and hence it has two possible meanings.
Tom needs a gym instructor like James.
Meaning 1: Tom needs a gym instructor who is like James: i.e. a gym instructor similar to James. Here, the compared entities are gym instructor and
James.

Tom needs a gym instructor like James.


Meaning 2: Tom needs a gym instructor as James does. Here, the compared entities are Tom and James. So, this meaning
indicates that both Tom and James need gym instructors.
Since this sentence does not convey one clear meaning, it is incorrect.
Why does this sentence convey an ambiguous meaning? What is the difference between this sentence and the first two
sentences?
In the first sentence there is only one logical comparison possible i.e. between president and Lincoln. The comparison
between the US and Lincoln is not possible so there is no ambiguity.
Similarly, in the second sentence there is only one logical comparison possible i.e. the comparison between Mike and a
professional guitarist. Since neither of them can be compared with guitar, there is no ambiguity.
Now, in the third sentence both the comparisons are logically possible, and hence it presents an ambiguous meaning.
4. Like my brother, I want to be an investment banker.
In the above sentence, the comparison is between I and my brother. My brother wants to be an investment banker, and so do
I. This same meaning can be conveyed if we say:
I want to be an investment banker, like my brother. (I compared with investment banker)
Lets take a couple more examples:
Unlike the honey bees, the masonry bees prefer to live in solitude.
Here, the comparison is between honey bees and masonry bees. While like shows similarity between two noun entities,
unlike shows the dissimilarity.
This sentence tells us that the masonry bees prefer to live in solitude. This characteristic is unlike the honey bees i.e. the honey
bees dont prefer to live in solitude.
Lisa takes care of her siblings like a mother does.
This sentence is incorrect since like is followed by a clause (a mother does). The correct way to write this sentence using like
is:
Lisa takes care of her siblings like a mother.
Lisa is compared to a mother. Lisa takes care of her siblings in the same way a mother does.

Now, try to apply this learning on the following official questions:


Like Auden, the language of James Merrill is chatty, arch, and conversational- given to complex syntactic flights as well as to
prosaic free-verse strolls.
A. Like Auden, the language of James Merrill
B. Like Auden, James Merrill's language
C. Like Auden's, James Merrill's language
D. As with Auden, James Merrill's language
E. As is Auden's the language of James Merrill
Unlike auto insurance, the frequency of claims does not affect the premiums for personal property coverage, but if the
insurance company is able to prove excessive loss due to owner negligence, it may decline to renew the policy.
A. Unlike auto insurance, the frequency of claims does not affect the premiums for personal property coverage,
B. Unlike with auto insurance, the frequency of claims do not affect the premiums for personal property coverage,
C. Unlike the frequency of claims for auto insurance, the premiums for personal property coverage are not affected by the
frequency of claims,
D. Unlike the premiums for auto insurance, the premiums for personal property coverage are not affected by the frequency of
claims,
E. Unlike with the premiums for auto insurance, the premiums for personal property coverage is not affected by the frequency of
claims,

TAKE AWAYS
1. When like + noun is separated from the sentence by a comma, the comparison is between the subject of the sentence and
the noun following like.
2. When like + noun is not separated from the sentence by a comma, the comparison is between the object of the sentence and
the noun that follows like. Note that this usage is correct only when there is no ambiguity about which two nouns are being
compared.
----------------------------------------------Evidence of/ evidence that:
Evidence of should be followed by a noun, and this noun should be the entity whose existence has been proved by the evidence.
For example:
Astronomers have discovered evidence of life on Mars. CORRECT
Scientists have discovered evidence of the dinosaurs being herbivores. INCORRECT
Scientists have discovered evidence that the dinosaurs were herbivores. CORRECT
In the first sentence above, astronomers discovered evidence about something: about life on Mars. This meaning is logically
correct. But in the second sentence, evidence is followed by the dinosaurs. Now, scientists did not discover evidence that
proved "the dinosaurs". The evidence they discovered showed them that the dinosaurs were herbivores. This meaning is not
brought out precisely in this sentence, since being herbivores is a modifier for dinosaurs. While this kind of usage is quite
common in everyday use, official questions are very precise, and the correct answers are extremely unlikely to allow this
imprecise use of an idiom. So, the third sentence makes the intended meaning far more precise by replacing evidence of with
evidence that, and following it up with a clause that conveys the meaning clearly.
Here are some more typical ways in which evidence of and evidence that are used:

Experts examining the power plant found no evidence of leaks.


The economy has shown evidence of a slowdown in the recent past.
The government has insisted that there is no evidence that the recent employment cuts were targeted at specific industries.
Geologists have discovered evidence that some of the glaciers in the Arctic have survived previous eras of global warming.

Examining how these sentences are constructed, we can see that evidence of is typically followed by a noun, whereas
evidence that is followed by a clause. So the usage of these terms depends on the intended meaning of the sentence. If we
mean that we have found evidence of a noun, we use evidence of. If we mean that we have found evidence of a particular
action, we use evidence that followed by a clause.
Try applying this understanding to these official questions:
1. Astronomers have uncovered evidence that a star that was as bright as the full moon exploding into view 340,000 years ago,
emitting dazzling radiation that could have disrupted Earth's protective ozone layer and sunburned our Stone Age ancestors.
A. that a star that was as bright as the full moon exploding into view 340,000 years ago, emitting
B. that a star as bright as the full moon exploded into view 340,000 years ago, emitting
C. of a star that was as bright as the full moon exploding into view 340,000 years ago and that it emitted
D. of a star as bright as the full moon, exploding into view 340,000 years ago and emitting
E. of a star as bright as the full moon that exploded into view 340,000 years ago and that emitted
2. Scientists have found new evidence of people initially registering emotions like sadness or anger in much the same way
asheartburnby monitoring what's going on within their bodies.
A. of people initially registering emotions like sadness or anger in much the same way as
B. of people initially registering emotions such as sadness or anger much the same as experiencing
C. that people initially register emotions such as sadness or anger in much the same way as they experience
D. that a person initially registers emotions such as sadness or anger much the same way as experiencing
E. that a person initially registers emotions like sadness or anger much the same as
I hope this understanding helps!

----------------------------------------------------------the root of a number cannot be negative

As it is with traditional pharmacies, on-line drugstores rely on prescriptions to be successful, since it is


primarily prescriptions that attract the customers, who then also buy other health related items.
A. As it is with traditional pharmacies, on-line drugstores rely on prescriptions to be successful
B. As with the case of traditional pharmacies, on-line drugstores rely on prescriptions to have success
C. As is the case with traditional pharmacies, prescriptions are the cornerstone of a successful on-line
drugstore - correct
D. As traditional pharmacies, so on-line drugstores rely on prescriptions to be successful
E. Like traditional pharmacies, the cornerstone of a successful on-line drugstore is prescriptions.
(a) "as it is with..." isn't parallel to anything. this is the main problem.
nothing "is with" online drugstores, so this choice is not sufficiently parallel.
b:
with the case of" is an incorrect rendering of "as is the case...".
(it's possible for "with the case of" to be correct, but the preposition "with" would have to make sense, AND you'd have
to be literally talking about "the case of" something.
for instance: the lawyer made history with the case of X, as with the case of Y
unlike this example, the problem in this thread doesn't satisfy either of these requirements)

in any case, you should know that '...is/was the case' has the authority to stand for an impressive
variety of things, including entire clauses. ('Do so' has the same impressive powers, especially vis-a-vis 'do
it', which is limited to situations featuring an actual 'it'.)
------------------------------------------------------------------------------------------------------------------------------------------------------------

Those skeptical of the extent of global warming argue that short-term temperature data are an inadequate means of
predicting long-term trends and point out that the scientific community remains divided on whether significant
warming will occur and what impact will it have if it does.
A. on whether significant warming will occur and what impact will it have if it does.
B. on whether warming that occurs will be significant and the impact it would have.
C. as to whether significant warming will occur or the impact it would have if it did.
D. over whether there will be significant warming or the impact it will have.
E. over whether significant warming will occur and what impact it would have.
ok, well: if choice a actually has the original wording posted at the beginning of this thread ('will it' instead of 'it will'),
then it's definitely wrong. 'will it' is only ok in the context of a question ('will it rain tomorrow?'), and can't be used as
a noun phrase.
process of elimination:
first, i hope it's clear that we want AND, not OR. according to the context of the problem, the scientific community is
divided on both of these issues (you don't get a choice between them), so 'and' makes more sense than 'or'.
that leaves choices b and e.
use PARALLELISM to resolve that dilemma:
choice b uses whether... and the impact in parallel.
choice e uses whether... and what impact... in parallel.
thus, choice e has better parallelism.
(incidentally, the same parallelism issue can also be used to get rid of answers c and d, the ones containing 'or')
hope that helps.
we can justify 'would' here by saying that it's a case of the subjunctive mood, which isn't often used in contexts like
this one. they're using 'would' instead of 'will' because the occurrence whose consequences are being considered is
hypothetical.
-----------------------------------------------------------------------------------------------------------------------------------------------------

An international group of more than 2,000 scientists project an average global warming that will be between
1.8 and 6.3 degrees Fahrenheit by the year 2000.
A. project an average global warming that will be between 1.8 and
B. project an average global warming to be from 1.8 to
C. project global warming that will average between 1.8 and
D. projects global warming to average from 1.8 to
E. projects an average global warming of between 1.8 and
well, d has two problems, at least:
- 'projects X to do Y' is unidiomatic. (you can just say 'projects NOUN', as is done in choice e, or you can say
something like 'projects that X will do Y').
- an average is a single data point, so there's no such thing as 'averag[ing] from 1.8 to 6.3'. on the other hand, it's quite
possible for a single data point to fall between two given values.
i've seen 'of between' in this sense before - think of the phrase 'between 1.8 and 6.3' as standing for a single number,
and parse the sentence accordingly (it reads as if it said 'average g.w. of 5 degrees', for instance) - so, if the original
poster has copied the problem correctly, we now know that the gmat accepts that construction.
correct usage of 'project':
'projects that X will do Y'
'projects NOUN'
'projects X to do Y' is unidiomatic.
-----------------------------------------------------------------------------------------------------------------------------------------------------------In a blow to those who still harbored the illusion that E-mail exchanges are private, a watchdog group recently uncovered a trick for
enabling an interloper to rig an E-mail message so that this person will be privy to any comments that a recipient had added as the
message is forwarded to others or sent back and forth.
(A) who still harbored the illusion that E-mail exchanges are private, a watchdog group recently uncovered a trick for enabling an
interloper to rig an E-mail message so that this person will be privy to any comments that a recipient had added
(B) who had still been harboring the illusion that E-mail exchanges are private, a watchdog group recently uncovered a trick for
enabling an interloper to rig an E-mail message so that this person was privy to any comments that a recipient might have added
(C) who still were harboring the illusion that E-mail exchanges are private, a watchdog group recently uncovered a trick enabling an
interloper to rig an E-mail message so that this person is privy to any comments that a recipient would add
(D) still harboring the illusion that E-mail exchanges are private, a watchdog group recently uncovered a trick that enables an
interloper to rig an E-mail message so that this person will be privy to any comments that a recipient might add
(E) still harboring the illusion that E-mail exchanges had been private, a watchdog group recently uncovered a trick that will enable
an interloper to rig an E-mail message so that this person was privy to any comments that a recipient might add
C:
a few things about choice c:
(1) 'who were still harboring' is wordy, esp. in comparison to the more concise version 'still harboring' presented in choice d**
(2) 'is privy' doesn't make as much sense as will be privy', because the comments to which the person will be privy haven't been
posted yet (it's a future occurrence)
(3) 'would add' can be interpreted in two ways: either as a subjunctive (for a hypothetical that isn't actually true, as in 'i wish you
dressed more nicely') or as the past-tense version of 'will'. the first doesn't make sense, because this hypothetical could clearly be
true, and the second doesn't make sense because the hack is being described in the present tense, not the past tense. 'might', a
present tense construction, makes more sense.
choice d, on the other hand, uses the more concise form 'still harboring' and the correct-tense forms 'will be' and 'might'
-------------------------------------------------------------------------------------------------------------------------------------------------------------

Critics of the research study claimed that monkeys are different enough from human beings that comparing
the two species canbe as questionable--or as useless--as comparisons between humans and mice.
(A) different enough from human beings that comparing the two species can
(B) different enough from human beings so that comparing the two species can
(C) different enough from human beings for comparing the two species to
(D) so different from human beings that comparisons between the two species can
(E) so different from human beings for comparisons between the two species to
C:
The author seeks to explain that comparing monkeys and humans is as useless as comparing humans and mice. Note
that the end of the sentence (the 2nd part of the comparison) states "as questionable as comparisons between humans
and mice". Thus the first part must discuss "comparisons" between monkeys and humans in order to maintain
parallel structure. Elminate A through C for using "comparing" instead of "comparisons"
---------------------------

Because Miranda, the smallest moon of Uranus, has a large number of different surface features, including craters,
mountains, valleys, and fractures, some astronomers suggest that at one time repeated impacts broke the surface
apart, and after which the fragments were subsequently rejoined because of mutual gravitational
attraction.
(A) repeated impacts broke the surface apart, and after which the fragments were subsequently rejoined because of
(B) repeated impacts on the surface broke it apart, after which the fragments having rejoined with
(C) through repeated impacts that the surface broke apart, after which the fragments subsequently rejoined by
(D) the surface broke apart with repeated impacts, after which the fragments having rejoined through
(E) the surface broke apart as a result of repeated impacts, after which the fragments rejoined through
A:
- don't need 'and' if you use 'after which' (no need for 2 linking words/phrases)
- redundancy: 'after which' means the same thing as 'subsequently', so having both is fatal
- 'because of' is sketchy; it suggests indirect causation, while the clear meaning is that gravitational attraction was directly
responsible
- bad parallelism: first half is in active voice, but second half is in passive voice
E:
CORRECT
- 'as a result of' = proper idiom
- proper parallelism: the surface broke apart is parallel to the fragments rejoined
-----"preposition + WHICH" plays the same modifying role as does plain "which" -- namely, it modifies the noun preceding the comma.
in the original sentence, then, this "after which" doesn't make any sense, because there is no noun in the appropriate position (i.e.,
preceding the comma).
in the correct answer, "after which..." correctly modifies the immediately preceding noun "(repeated) impacts".
-----------------------

Diesel engines burn as much as 30% less fuel than gasoline engines of comparable size, as well as emitting far less
carbon dioxide gas and far fewer of the other gases that have been implicated in global warming.
A. Same
B. of comparable size as well as emit far less carbon dioxide gas and far fewer of the other gas having
C. of comparable size, and also they emit far fewer carbon dioxide and other gases that have
D. that have a comparable size , and also they emit far less carbon dioxide and other gases that have
E. that have a comparable size, as well as emitting far fewer carbon dioxide and other gases having.
you could have "fewer gases" (= not as many different kinds), or "less OF the gases" (= a smaller total quantity)... but
not "less gases".
ques:
In answer choice A, I do not understand how burn and emitting can be parallel to each other. Please explain.?

ans:
they aren't. from this example, you can learn that 'as well as' is not strictly a parallel marker.
------------------------------------------

Greatly influenced by the Protested missionary Samuel Kirkland, the Oneida was the only one of the fivenation Iroquois League who sided with the colonists during the American Revolution.
A. was the only one of the five-nation Iroquois League who sided
B. was alone of the five-nation Iroquois League when they sided
C. alone among the five-nation Iroquois League sided
D. were the only ones out of the five nations of Iroquois League in siding
E. only of the five-nation Iroquois League had sided
ah yes, a tricky one.
'The Oneida' is plural, in much the same way as 'the English' or 'the French' would be plural (the French eat foods that are rather
bizarre by the standards of most other countries). that observation knocks off choices a and b.
choice d is wrong because it is extremely, incredibly wordy. 'out of' is also a problem (because the gmat would only use 'out of' if it
meant literally out of something, or in standard idioms like '3 out of 4'), but the wordiness of this choice should smack you in the
face the second you look at it.
choice e is wrong because it uses the past perfect to refer to a single event. you need the simple past here (just 'sided', not 'had
sided'), because there is no more recent time signal to justify the use of the past perfect for the 'earlier' event.
that leaves choice c, which uses the simple past correctly and is concise. (notice that singular/plural isn't an issue with that choice,
because past tenses, with the exception of was/were, don't conjugate for number)
-------------------------------------

The human nervous system and a telephone system superficially resemble each other, not only because
the former carries information in the form of electrical impulses and because all of its neural pathways
converge in the brain and spinal cord, which together form a kind of central exchange.
(A) The human nervous system and a telephone system superficially resemble each other, not only
because the former carries
(B) The human nervous system and a telephone system bear a superficial resemblance because they both
carry
(C) The human nervous system bears a superficial resemblance to a telephone system both because the
former carries
(D) Superficially, a telephone system resembles the human nervous system both because they carry
(E) There is a superficial resemblance between a telephone system and the human nervous system, not
only because they both carry

---------------------------------------------------------------------------

Before scientists learned how to make a synthetic growth hormone, removing it painstakingly in small amounts from
the pituitary glands of human cadevers.
A)
B) scientists had learned about making a synthetic growth hormone, they had to remove it painstakingly.
C) scientists learned how to synthesize the growth hormone, it had to be painstakingly removed
D) learning how to make a sythetic growth hormone, scientists had to remove it painstakingly
E) learning how to synthesize the growth hormone, it had to be painstakingly removed by scientists

In the problem at hand, the original sentence, along with answer choices B and D, indicate that scientists removed
"a syntheticgrowth hormone" from cadavers. This is nonsensical, as something synthetic, by definition, does not come
from a natural source. Thus, A, B, and D can be eliminated based on their lack of clarity.
choice e is wrong, because it starts out with an initial modifier lacking a subject; such modifiers MUST modify the
noun IMMEDIATELY FOLLOWING the comma. in the case of choice e, this would mean that 'it' (referring to the
hormone itself) learned how to make the hormone. that's crazy
---------------------------------

Some anthropologists regard the early hominids' manner of walking as being less efficient than in modern human
beings.
A as being less efficient than in
B as less efficient than it is in
C as less efficient than that of
D to be less efficient than that of
E to have been less efficient than it is in
choice d:
- bad idiom: you can't say 'regard X to be Y'
'that of' (singular) and 'those of' (plural) are possessive constructions that allow us to write parallel comparisons
without the bothersome repetition of specific nouns.
for instance:
Beethoven's symphonies were more revolutionary than Bach <-- wrong, because music is being compared to a
person
Beethoven's symphonies were more revolutionary than Bach's symphonies <-- correct, but with bothersome
repetition
Beethoven's symphonies were more revolutionary than Bach's <-- ok according to most sources, but i've never seen
the gmat use the apostrophe+s for a plural
Beethoven's symphonies were more revolutionary than those of Bach <-- the best way to write this sentence.
in the last of these, 'those of bach' = 'bach's symphonies'.
-----------------------------------------------------

The single-family house constructed by the Yana, a Native American people who lived in what is now northern
california, was conical in shape, its framework of poles overlaid with slabs of bark, either cedar or pine, and banked
with dirt to a height of three to four feet.
a. banked with dirt to a height of
b. banked with dirt as high as that of
c. banked them with dirt to a height of
d. was banked with dirt as high as
e. was banked with dirt as high as that of
a final problem with choice d is that the phrase 'as high as' should be followed by one value, not a range.
some of our players weigh as much as 300-325 pounds --> bad phrasing
some of our players weigh as much as 325 pounds --> good phrasing
--------------------------Performing a risky maneuver that required precision flying, not only did space shuttle astronauts retrieve an orbiting
satellite, it was done simultaneously while avoiding being rear-ended by a passing ultraviolet telescope.
A. not only did space shuttle astronauts retrieve an orbiting satellite, it was done simultaneously while avoiding
B. not only was an orbiting satellite retrieved by space shuttle astronauts, but they also simultaneously avoided
C. an orbiting satellite was retrieved by space shuttle astronauts who also avoided simultaneously

D. space shuttle astronauts retrieved an orbiting satellite, simultaneously while avoiding - INCORRECT
E. space shuttle astronauts retrieved an orbiting satellite and simultaneously avoided
D:
the placement of "simultaneously" is problematic. it describes "avoid"... so it should appear directly next to "avoid", as
it does in the correct answer.
incidentally, if "simultaneously" and "while" were reversed, then D would become a respectable sentence
E has objectively better placement; D has objectively worse placement. so, D is gone and E stays.

Three out of every four automobile owners in the United States also own a bicycle.
A Three out of every four automobile owners in the United States also own a bicycle.
B Out of every four, three automobile owners in the United States also owns a bicycle.
C Bicycles are owned by three out of every four owners of automobiles in the United States.
D In the United States, three out of every four automobile owners owns bicycles.
E Out of every four owners of automobiles in the United States, bicycles are also owned by three.
ques:
I eliminated Choice A because I thought the subject "three out of every four auto owners" requires a plural object "bicycles"
instead of "a bicycle."?
ans:
No. In constructions like this one, it's understood that "a bicycle" refers to a bicycle for each owner.
That's not unique to this sentence:
Most students at this university ride a bicycle to class.
It's December, so 9 out of 10 New Yorkers will be wearing a black overcoat.
etc.
The context of these sentences is pretty obvious: one item per person.

"three" + modifiers. so, plural.


of every 4 days, 3 are sunny and 1 is rainy.
1 of every 4 days is rainy.
3 of every 4 days are sunny.
The problem with C is that it becomes a statement about bicycles, and thus takes on an unreasonable
meaning.
Here's what I mean: Consider the following two statements.
1/
Teenage girls are concerned with fashion.
2/
Fashion is a concern of teenage girls.
The first statement is about teenage girls. I.e., according to #1, teenage girls in general are concerned with
fashion, i.e., it's relatively rare to find a teenage girl who is unconcerned with fashion.
#1 says nothing about whether other demographics (e.g., teenage boys, middle-aged adults) are concerned
with fashion.
The second statement is about fashion. According to #2, fashion is largely confined to teenage girls, and

it's rare to find someone else (a boy or an adult) who is concerned with fashion.
#2 says nothing about whether most teenage girls are concerned with fashion.
Note the wide divergence between the two. In fact, #1 is basically true while #2 is false.
That's the problem with C: it implies that the ownership of bicycles is largely confined to people who also
own cars. Weird.
(You can also eliminate it because it's needlessly indirect, but that's the real reason.)
=--------------------------------------------------------In the mid-1920s the Hawthorne Works of the Western Electric Company was the scene of an intensive
series of experiments that would investigate changes in working conditions as to their effects on workers'
performance.
(A) that would investigate changes in working conditions as to their effects on workers' performance
(B) investigating the effects that changes in working conditions would have on workers'performance
(C) for investigating what the effects on workers' performance are that changes in working conditions
would cause
(D) that investigated changes in working conditions' effects on workers' performance
(E) to investigate what the effects changes in working conditions would have on workers'performance
(b) is the best choice here.
(a) is vague because it's overly indirect: the meaning of "investigate changes ... as to their effects" is unclear. what's more, it's
probably considered unidiomatic as well, at least in this sort of context.
(b) = correct
the participle "investigating" follows "experiments" immediately. no filler words are necessary; this is good concision.
the wording is clear; there are no awkward double possessives, etc., as in some of the other choices.
"would" is used properly here, as a past-tense form of "will". (i.e., if this sentence were translated into the present tense, it would
read "...that changes ... will have")
(c) is ridiculously wordy; there's no way you should give this choice any serious consideration. if you don't realize pretty quickly that
this choice is wrong, you should go back and read through a bunch of correct OG answers, trying to internalize the sights and sounds
(the "vibe") of the correct answers.
(d) "changes in working conditions' effects" is at best awkward and vague, and at worst ambiguous: the intended meaning is the
effects of the changes, but this sentence seems to indicated the effects of the conditions themselves. in other words, a literal reading
of this sentence seems to indicate that the conditions themselves haven't changed - only their effects have. that's not the intended
meaning of the original.
(e) "what the effects" is ungrammatical.
also, in constructions of this sort, "what" is generally redundant / unnecessary; it's better merely to say "to investigate X" rather than
to say "to investigate what X is" (or other such wordy construction).

--------------------------------------------------------------------------------Nine months after the county banned jet skis and other water bikes from the tranquil waters of Puget
Sound, a judge overturned the ban on the ground of violating state laws for allowing the use of personal
watercraft on common waterways
A same
B of their violating state laws to allow
C that it violates state laws that allowed - incorrect
D that it violated state laws allowing - correct
E that state laws were being violated allowing
choice (c) is wrong because the tenses don't make sense. 'violates' is in the present tense, but 'allowed' is
in the past tense. either one of these tenses could potentially make sense individually, but the combination
is absurd: you can't violate (present tense) a law thatused to allow something (past tense). if you're going

to violate the law in the present tense, then whatever part of the law was violated had better carry over
into the present tense.
interestingly, all 3 other tense combinations make sense: violates/allows, violated/allows (if the law is still
in effect), and violated/allowed (if the law is no longer in effect).
choice (d) circumvents this issue altogether by employing the participle form (-ing). despite its name (it's
formally called the "present participle"), this form is NOT necessarily a present-tense construction; rather,
it has no inherent tense at all, and merely adopts the tense of whatever verbs in the sentence do have a
tense. therefore, in choice (d), 'allowing' takes place in the past tense, simultaneously with 'violated'.
-----------------------------Ozone, a special form of oxygen that screens out harmful UV rays, reaches high concentrations twelve
miles above Earth, where it has long appeared that it was immune from human influence; we have now
realized, though, that emissions of industrial chloroflorocarbons deplete the ozone layer.
A) has long appeared that it was immune from
B) has long appeared to have been immune from - INCORRECT
C) has long appeared as being immune to
D) had long appeared immune to
E) had long appeared that it was immune to
The verb tense in the underlined part of the sentence must be compared with what is taking place now. As
the sentence states wehave realized that emissions of industrial chloroflorocarbons deplete the ozone
layer (present perfect), we need the past perfect had appeared to indicate a time period prior to the
present. Additionally, "immune from" means free of exempt from (eg. immune from prosecution),
whereas "immune to" means not susceptible (eg. immune to chicken pox). In this case, the ozone layer
was considered not susceptible to human influence. Thus, "immune to" is idiomatically correct
WHY SIMPLE PAST NOT REQUIRED:
You don't need an actual past-tense verb. You just need a clear indicator of a definite point in the past
the point that serves as the point of view for "had ___ed".
In this sentence, it's clear that, at present, ozone does not appear "immune to xxxxx".
So, the context is this:
At some point in the past, people thought that ozone was immune to xxxx. For some time up to that
point, it had appeared that way.
Now, we know this isn't true anymore.
Same thing with "By 1990, all of Cynthia's granddaughters had gotten married". 1990 isn't a past-tense
verb, but it's plenty good enough as a past reference point for "had ___ed".
---------------------------------------------------So-called green taxes, which exact a price for the use of polluting or nonrenewable fuels, are having a
positive effect on the environment and natural resource base of countries as varied as China, the
Netherlands, and Hungary.
a. as varied as - CORRECT
b. as varied as are
c. as varied as those of
d. that are as varied as - INCORRECT
e. that are varied as are

D:
There's a very subtle difference in meaning, although Stacey is correct that the main issue is that of
redundancy/wordiness. But consider the difference between these two sentences, which would illustrate
the point:
(1) I get mail from cities as far away as Providence.
(2) I get mail from cities that are as far away as Providence.
If I'm in San Francisco, then sentence #2 means, strangely enough, that I get mail from cities that are all
exactly 3,082 miles away (the distance from SF to Providence). Sentence #1 implies no such thing.
Similarly, the wrong answer (D) seems to imply that EACH country is somehow as 'varied' as EACH other
country. That's not the intended meaning, which is that the SET of countries is varied.
when you have a COMPARISON construction such as the one here ("as varied as..."), it must refer to
whatever noun, noun phrase, or other type of construction IMMEDIATELY follows.
-------------------------New equipment and other improvements reduced the amount of time"from eleven hours in 1982 to six in
1988"needed by workers so that they could produce a ton of steel.
A. needed by workers so that they could produce
B. needed when workers are producing
C. workers need in producing
- present tense doesn't make sense
D. workers needed to produce
E. workers need for their producing of

------------------------------------------The loan company announced it would soon lend money to borrowers with proven records of their not
paying back their loans on time, collectively known as the subprime lending market.
A. Of their not paying back their loans on time,
B. Of not paying back their loans on time, a group
C. Of not paying back their loans on time, with such a group
D. That they do not pay back their loans on time,
E. That they do not pay back their loans on time, such a group
(d) and (e) are unidiomatic; you can't write "proven records that they..."
---------------------------------------------United States Senator Daniel Inouye was appointed to several posts within the Democratic party during
his first term, that included assistant majority whip and vice-chair of the Democratic Senatorial
Committee.
A. that included

B. which includes
C. including
D. some of which were
E. among them being
if this is really a gmat problem, then the quality of those problems is declining at an alarming rate (or the
problem writers were asleep, drunk, etc. when they wrote this problem ... ugh)
--------------------------The number of people flying first class on domestic flights rose sharply in 1990, doubling the increase
of the the previous year.
a. doubling the increase of
b. doubling that of the increase in
c. double as much as the increase of
d. twice as many as the increase in
e. twice as many as the increase of
B:
there's no noun for which "that" can stand. you're trying to make "that" stand for "increase", but,
unfortunately, "increase" isn't there.
by the way, you can strike (d) and (e) without even seeing the prompt.
as soon as you see "twice as many as the increase...", these choices are automatically incorrect (since
"increase" is not a countable quantity).
Doubling the increase: this is the adverbial modifier.( COMMA + VERB ING)
in this case, you can't use "..., twice as many as...", because that's an appositive modifier. appositives must
modify some noun that comes IMMEDIATELY before the comma, which in this case would have to be
whatever figure is twice whatever other figure. since no such figure is given, you can't use this
construction.
Ques:
'Twice' is actually an adverb. Doesn't 'twice - followed by a comma' work as a adverbial modifier and
therefore will modify the entire clause preceeding the comma?
ans:
if you want, you can just memorize this an idiom, but "twice Y" / "X times Y" can definitely be used as an
appositive.
here's the basic idea:
if you say "twice as many", then this construction should be paired with a countable noun.
e.g., twice as many dogs --> "dogs" is a countable noun
if you said "twice as much", then this construction should be paired with an uncountable noun.
e.g., twice as much water --> "water" is an uncountable noun
if the noun in question is already an explicitly numerical quantity, then you should
use neither "much" nor "many". instead, you should just use "twice" or "double" by itself.
e.g., twice the increase --> "increase" is an explicitly numerical quantity

these rules are followed pretty closely.


so, for instance:
twice as much water --> correct, since "water" is an uncountable noun (but is not an explicitly numerical
quantity)
twice the water... --> incorrect, since water is not a numerical quantity
twice as much as the increase... --> incorrect; redundant
twice the increase... --> correct
increase in / increase of:
genuine split. not red herring.
"the increase in X" means that X itself has increased. therefore, "the increase in the previous year" doesn't
make sense, because the implication would be that the previous year itself had increased (what would that
possibly mean?).
"the increase of TIME PERIOD", by contrast, means exactly what it should mean in this particular
instance.
------------------------------------------As the former chair of the planning board for 18 consecutive years and a board member for 28 years,
Joan Pilkin attended more than 400 meetings adn reviewed more than 700 applications.
A-As the former
B-The former
C-Former
D-She was
E-As the- correct
meaning based question
An individual would not be the former chair of a planning board for 18 consecutive years; instead, the
sentence is attempting to say that Joan was actully the chair of the planning board for 18 consecutive
years and that she attended these meetings during those 18 years. Thus, choices A, B, and C and be
eliminated. As D is a run on sentence, E is the best answer.
----------------------------------------------------------------Many environmentalists, and some economists, say that free trade encourages industry to relocate to countries with
ineffective or poorly enforced antipollution laws, mostly in the developing world, and that, in order to maintain
competitiveness, rich nations have joined this downward slide toward more lax attitudes about pollution.
A. that, in order to maintain competitiveness, rich nations have joined this downward slide toward more lax attitudes
about pollution - correct
B. that, for maintaining competitiveness, rich nations join in this downward slide toward more lax attitudes
about pollution
- the word ''join'' changes the meaning.
C. that rich nations join this downward slide toward more lax attitudes about pollution because of wanting to
maintain competitiveness
D. that in rich nations, joining this downward slide toward more lax attitudes about pollution is a result of wanting to
maintain competition

E. that wanting to maintain competition is making rich nations join in this downward slide toward an attitude about
pollution that is more lax
---------------------------------------------------------------------------------------------------------if "after" is followed by a verb form, that verb form must be a gerund (-ING). hence "after pounding".
----------------------------------------------------------It is unclear whether chimpanzees are unique among nonhuman species in their ability to learn behaviors from one another, or if,
when other animals are studied in as much depth, similar patterns would be found.
(A) if, when other animals are studied in as much depth, similar patterns would be found
(B) if other animals were studied with as much depth they would exhibit similar patterns
(C) would similar patterns be found in other animals if they were studied in as much depth
(D) whether similar patterns would be exhibited in other animals that were studied with as much depth
(E) whether other animals would exhibit similar patterns if they were studied in as much depth
Ron:
the sentence talks about something that is still unclear.
if there were "animals that were studied"-- in the past-- then we would already know the results of those studies, an idea that
contradicts the rest of the sentence.
note that "were studied" is sometimes a hypothetical form, rather than a past-tense form. in that form, it would be reasonable here.
We don't know what would happen if these animals were studied in such depth.
in fact, this is how the correct answer works.
choice D, on the other hand, postulates the existence of "animals that were studied". that's not a hypothetical; it asserts that there
are actually animals that have already been studied in the manner described.
-------------------------------Paper production accounts for approximately 40 percent of the world's industrial use of wood, and the market for paper is growing
faster than it is for all major wood products.
A. the market for paper is growing faster than it is for all
B. the market for paper is growing faster than the market for all other
C. the market for it is growing faster than it is for all other
D. its market is growing faster than the market for all
E. its market is growing faster than it is for all other
the pronouns "it" and "they" (along with other cases of those pronouns, such as "its", "their", "them") always refer to the
intended noun plus any essential modifiers attached to that noun. so, in choice (a), the pronoun "it" erroneously refers to
"the market for paper". this is an error in meaning.
note that "it"/"its" is a faulty pronoun in the last three choices; in those three choices, the intended referent of "it" is paper, but
there's a problem: the noun paper isn't present in this sentence. (in the phrase "paper production" , paper is an adjective.
--------------------------------------------------------------------------------

Broccoli thrives in moderate to cool climates and propagated by seeds, either sown directly in the
field or in plant beds designed for producing transplants.
(A) propagated by seeds, either sown directly in the field or in plant beds designed for producing
(B) propagated by seeds, sown either directly in the field or sown in plant beds designed so as to produce
(C) is propagated by seeds either sown directly in the field or in plant beds designed so that they produce
(D) is propagated by seeds sown either directly in the field or in plant beds designed to produce
(E) is propagated by seeds, sown either directly in the field or sown in plant beds designed for producing
ques:
I recall from a previous GMATPrep problem(Both weakened by concern about the government`s
agreement...) that the inclusion of additional modifiers/adjectives/adverbs does not affect parallelism.

In this example parallel elements are "in the field" and "in plant beds".
"directly" is an adverb and does not affect parallelism.
Is that correct understanding?
ans:
absolutely perfect understanding.
correct ans: D
-----------------------------------------The cottontail rabbit population in Orange County, California, has increased unchecked in recent years as a result of the removal
ofthe native fox population and the clearing of surrounding woodlands.
(A) years as a result of the removal of
(B) years as a result of removing
(C) years, resulting from the removing of
(D) years, which is a result of removing
(E) years, which is a result of the removal of
removal is better.
there are a bunch of words like these, with specific noun forms that, simply because they exist, are considered better than the
corresponding gerund (-ing) forms. notable among this class of nouns are the '-al' forms, like removal, betrayal, etc.
unfortunately, there are no general tips of the form you're soliciting; such are the joys of the english language. the closest i can get to
a general rule is this: if a special noun form exists, use it. if not, use the -ing form.
as for your examples above, you wouldn't want 'removing' because removal is a better substitute. clearing, though, is fine because
there's no specially designated noun form for that one.
(1)
you can't use 'resulting from' as an adverbial modifier, as is done here. in general, 'resulting from' is only used as an adjective
modifier, almost always without a comma, as in
the pollution resulting from the chemical spill forced all the local residents to evacuate.
-------------------------------------------------------

Archaeologists in Egypt have excavated a 5,000-year-old wooden hull that is the earliest surviving
example of a "built" boat"in other words, a boat constructed out of planks fitted together-and that thus
represents a major advance, in terms of boat-building technology, over the dugout logs and reed vessels of
more ancient vintage.
A. together-and that thus represents
B. together-and this has represented
C. together, and it represents
D. together that was representing
E. together to represent
the usage of long dash:
The long dash is used to indicate an example or an aside. Only the example or aside goes inside the dash, so that example either has
to go all the way to the end of the sentence or we need to "close" the dash at some point by putting a second one in. In this case, the
"in other words" text only applies to the "built boat" concept, so I want to "close" the dash when I'm done talking about it. So
eliminate C, D, and E.

these are called nouns with identical singular and plural. Fortunately, there aren't many of them!
You've spotted one to watch out for. Also beware of:

"series"
"aircraft" (or "hovercraft," "watercraft," or any kind of -craft vehicle)
lots of animals, including "deer," "buffalo," "moose," and many types of fish

species

--------------------------Is species a collective noun

no, although it's one of those nouns for which the singular and plural have the same form.
1 species, 2 species, 3 species, etc.
---------------------------------------------------------Unlike frogs that metamorphose from tadpoles into adults within a one-year period, it takes three to four years for the mountain
yellow-legged frog of the Sierra Nevada to reach adulthood, and so they are restricted to deeper bodies of water that do not dry up
in summer or freeze solid in winter.
a/ it takes three to four years for the mountain yellow-legged frog of the Sierra Nevada to reach adulthood, and so they are
b/ it takes the mountain yellow-legged frog of the Sierra Nevada three to four years until it reaches adulthood, and therefore it is
c/ in the Sierra Nevada, mountain yellow-legged tree frogs take three to four years to reach adulthood, thus being
d/ mountain yellow-legged frogs of the Sierra Nevada take three to four years until they reach adulthood, thus ( restricted)
e/ mountain yellow-legged frogs of the Sierra Nevada take three to four years to reach adulthood, with the result that they are

two things:
1/
that kind of modifier normally describes the closest action. so, you have "(thus) restricted..." trying to modify "...to reach adulthood".
this is problematic, since that modifier doesn't have anything to do with reaching adulthood; it describes a situation that
obtainsbefore the animal reaches adulthood.
2/
"until" describes something that happened / was true up to a certain point in time. (e.g., Charlie was very weak until he started
lifting weights.)
"take 3-4 years" is not such a thing.
-----------------------------------------------------less -> Uncountable
lower -> Countable

-------------------

In some species of Cricket, the number of chirps per minute used by the male for attracting females rise and fall
in accordance with the surrounding temperature, and they can in fact serve as an approximate
thermometer
A. for attracting females rise and fall in accordance with the surrounding temperature, and they can in fact serve
B. for attracting females rises and falls in accordance with the surrounding temperature, which can in fact serve
C. in attracting females rise and fall in accordance with the surrounding temperature, in fact possibly serving
D. to attract females rises and falls in accordance with the surrounding temperature, and it can in fact serve
E. to attract females rises and falls in accordance with the surrounding temperature, in fact possibly serving
fight is b/w d and e:
3 differences:
1). can- capacity
possibly- might
changes the meaning
2). check the original sentence:
there are 2 clauses which are connected by ''and'':
but E changes the meaning by making it dependent on previous clause:

3).''

in fact possibly serving'' is awkward.

--------------------------------------------------------------------------

A decade after initiating the nation's most comprehensive and aggressive antismoking program, per capita
consumption of cigarettes in California declined from over 125 packs annually per person to about 60, a drop more
than twice as great as in the nation as a whole.
A) per capita consumption of cigarettes in California declined from over 125 packs annually per person to about
60, a drop more than twice as great as
B) annual per capita consumption of cigarettes in California declined from over 125 packs to about 60, more
than twice as great as that
C) California's annual per capita consumption of cigarettes declined from over 125 packs per person to about 60,
more than twice as great as the drop
D) California has seen per capita consumption of cigarettes decline from over 125 packs annually to about 60, a drop
more than twice as great as that
E) California has seen annual per capita consumption of cigarettes decline from over 125 packs per person to about
60, more than twice as great as that
''per capita'' and '' per person'' redundant.
this is a very common error, so be sure that you can recognize it at once.
this sentence starts with a 'headless modifier' (headless in the sense that it doesn't have a subject). in sentences starting with such
modifiers, the modifier MUST modify the noun immediately following the comma.
so, choice b implies that 'per-capita consumption of cigarettes' initiated the nation's most comprehensive antismoking program
(clearly an absurdity). choice d, by contrast, correctly states that california initiated the program.
notice that the same consideration also eliminates choice a (which states that consumption initiated the program) and choice c
(which states that CA's consumption initiated the program).
E:
The problem with E is not the placement of "annual", but the failure to include a word like "drop". The modifier (the part after the
comma) is "more than twice as great as that". But what is more than twice as great as what? It is not the number of packs which is
more than twice as great, but the drop in the number of packs. Because this is not stated in E, it is wrong.
QUES:

Could you explain why the verb here is "decline" not "declines"?
Ans:
Ron:
if you still don't see how this works, use the same sort of construction in a much simpler context:
i watched my favorite team won --> incorrect
i watched my favorite team win --> correct
''a drop'' is missing from A,C,E which is incorrect. If this word is missing , then modifier will refer to immediate preceding noun.
i.e. 60.

--------------------------------------To meet the rapidly rising market demand for fish and seafood, suppliers are growing fish twice as fast
as their natural growth rate, cutting their feed allotment by nearly half and raising them on special
diets.
A their natural growth rate, cutting their feed allotment
B their natural growth rate, their feed allotment cut
C growing them naturally, cutting their feed allotment
D they grow naturally, cutting their feed allotment
E they grow naturally, with their feed allotment cut.

A:
In this case, you have a problem of redundancy:
a RATE can't be FAST.
* the rate can be high;
* the increase itself can be fast.
similarly,
a height can't be tall (a person can be tall, or a height can be greater than...)
a bank account can't be rich (a person can be rich, or a bank account can contain a large amount of money)
etc.
lest you think this is just a matter of excessive nitpicking on our part, this difference is corroborated by several official problems.

C:

growing them naturally doesn't make much logical sense. You 'let them grow naturally' , i.e., without intervention.

---------------------------------------------------

Comparisons:
Dirt roads may evoke the bucolic simplicity of another
century, but financially strained townships point out
that dirt roads cost twice as much as maintaining
paved roads.
(A) dirt roads cost twice as much as maintaining
paved roads
(B) dirt roads cost twice as much to maintain as
paved roads do
(C) maintaining dirt roads costs twice as much as
paved roads do
(D) maintaining dirt roads costs twice as much as it
does for paved roads
(E) to maintain dirt roads costs twice as much as
for paved roads
Comparisons are comparisons. The whole point is that you're comparing things that can legitimately be compared. So, unless the
sentence explicitly indicates otherwise, comparison sentences automatically compare things that are actually comparable to
each other.
E.g.,
French bulldog puppies cost more than Samoyed puppies.
French bulldog puppies cost more than Samoyed puppies do.
^^ These sentences discuss the cost of purchasing the puppies themselves.
French bulldog puppies cost less to feed than Samoyed puppies.
French bulldog puppies cost less to feed than Samoyed puppies do.
^^ These sentences discuss the cost of feeding the dogs, not the cost of purchasing them.
You do need a certain degree of parallelism. (E.g., here, "do" needs a verb, "cost", to which it can be parallel.) Beyond that, though,
it's mostly just common sense.
If for some reason you wanted to compare the cost of feeding one of the puppies with the cost of purchasing the other one -- for
whatever reason -- then you'd have to write a sentence that's very specific about that intention.
E.g., It costs more to purchase a French bulldog puppy than to feed a Samoyed for ten years

from Brian :
1) The two things compared must be compared in equivalent form.
Here, we could compare:
Dirt roads to paved roads
Maintaining dirt roads to maintaining dirt roads
But comparing "the cost of maintaining dirt roads" to "paved roads" is incorrect - one is a cost, and the other is a road...they could never be alike!
Make sure that, when a comparison is drawn, you check to ensure that the two items are in equivalent form. I like to envision a balance scale from
chemistry class as a mental picture. If I'm weighing a substance in a petri dish I must account for the weight of the dish on the other side of the
balance! Similarly, if I'm comparing a cost of one item, I have to make sure I compare it directly to the cost of the other.

2) Comparison idioms should be in the right form.


This one doesn't have a mistake, but you should get in the habit of seeing:
"As Many As" or "As Much As" ---> Equality
"So Many That" or "So Much That" ---> Critical Mass (e.g. "there is so much pollution in the air that we can't go outside")
"More Than" or "Less Than" ---> Inequality
An easy way for the testmakers to write a wrong-but-tricky answer is to criss-cross these idioms (e.g. "As many that" or "More...as")
In this case, the comparisons are all off but one:
A) Dirt roads cost vs. Maintaining paved roads
B) Dirt roads cost vs. paved roads do ("do" takes the place of "cost") ---> CORRECT!
C) Maintaining dirt roads costs vs. paved roads cost
D) Maintaining dirt roads costs vs. it does
E) To maintain dirt roads vs. for paved roads

------------------------------------For every A, B:
Here, the word for is a preposition, so A and B have to be nouns. In fact, in this idiom, A & B have to be
concrete nouns. This idiom discusses a correspondence, expressing the ratio between the elements of A and the
elements of B. It is frequently used in economic and political contexts. In fact, this is a frequent idiom in GMAT
math problems.

8) For every dollar Roscoe Corporation spends on R & D, Utica Central spends seven dollars.
9) For every vote McCormick wins in the Midwest with this new strategy, he stands to lose two or three in the
Northeast and in California.
10) For every 10% increase in the value of x, y increases 25%.

Not so much A as B:

The final one is the most sophisticated of these five: not so much A as B. It demonstrates a difference in degree:
whatever is being asserted, A is true or relevant, but it is less true or less relevant, and B is more so by
comparison. This is used for nouns primarily for nouns, noun-like phrase (infinitives & gerunds), prepositional
phrase or participial phrases.
2) The CEO wants to organize a new division around these six products, not so much to promote the sales of these
six as to establish a foothold in a new market sector.
3) In composing the Bill of Rights, the Founding Fathers valued not so much defending the rights of criminals as
protecting any innocent person from unjust punishment.
4) Ironically, Columbus is remembered not so much for his original goal, finding the sea route to Asia, as for his
accidental discovery, North America.
5) After his dramatic home run in the 1988 World Series, Gibson rounded the bases not so much running as
hobbling.

----------------------------------------------Retailers reported moderate gains in their November sales, as much because of their sales of a year earlier being so bad as
that shoppers were getting a head start on buying their holiday gifts.

A. of their sales of a year earlier being so bad as that


B. of their sales a year earlier having been as bad as because
C. of their sales a year earlier being as bad as because
D. their sales a year earlier had been so bad as because
E. their sales of a year earlier were as bad as that

* improper parallelism (as much because X as that Y)


* "as bad as" sets up a comparison, which doesn't have a second half in this case (you can't just say "as bad" by itself; you have to
mention ... as bad as what else. by contrast, "so bad" can be used by itself, as it is in the correct answer here; "so ADJ" is not a
comparison.)
----------------------------------

For members of the seventeenth-century Ashanti nation in Africa, animal-hide shields with wooden frames were essential items of
military equipment, a method to protect warriors against enemy arrows and spears.
(A) a method to protect- no method is described in this sentence.
(B) as a method protecting
(C) protecting- correct
(D) as a protection of
(E) to protect
Ron:
"...were essential items..." is not an action undertaken with the purpose/goal of "protecting" the warriors. rather, "protecting
the warriors" IS a (more detailed) statement of exactly how these items were "essential".
-----------------------------------------Comma + ___ing doesn't necessarily have to give a consequence of the preceding clause (though that is one way in which it's often
used). As long as it gives some sort of description/clarification/illustration/consequence/etc. of the preceding clause, you're fine.
------------------------GMAC frequently places an adverb between a helping verb and the verb being helped. The following examples are all from OAs in
the OG12:
are IN EFFECT told
have NEVER been sighted
can HARDLY be said
can CONSTANTLY change
have RECENTLY discovered
were EVENTUALLY exhibited
were FIRST aired
can QUICKLY analyze
would LATER make
are CLOSELY related
has ALREADY stimulated
is WIDELY accepted
Thus, would IN THE PAST have conceded is in keeping with GMAC's rules.
------------------------------------------------------------------------Although various eighteenth and nineteenth-century American poets had professed an interest in Native American poetry and had pretended to
imitate Native American forms in their own works, until almost 1900,scholars and critics did not begin seriously to study traditional Native American
poetry in native languages.
A) until almost 1900,scholars and critics did not begin seriously to study
B) until almost 1900 scholars and critics had not begun seriously studying
C) not until almost 1900 were scholars and critics to begin seriously to study
D) it was not almost until 1900 when scholars and critics began to seriously study

This choice changes the modifier and it is not clear what almost modifies. Per the intended meaning almost
should modify 1900 implying around 1900 e.g. it could be in 1899 or 1901. But placing almost before until no
longer communicates the intended approximation of time period around 1900.
E) it was not until almost 1900 that scholars and critics seriously began studying

Modifiers until almost 1900 = Incorrect


But, since it is separated by commas, it can either modify Clause 1 action or Clause 2 action. And hence modification
is ambiguous.

dataisplural,datumissingular.
media=plural
singular(medium)
Similartodata/datum

whereasAcanthis,Bcannot.
incaseof''whereas'',AandBneedtobe||el.

declaredABcorrect
declaredBAcorrect
DECLAREDAasBincorrect

Seldommorethat40feetwideand12feetdeep,butitran363milesacrosstherugged
wildernessofupstateNewYork,theErieCanalconnectedtheHudsonRiveratAlbanyto
theGreatLakesatBuffalo,providingtheportofNewYorkCitywithadirectwaterlink
totheheartlandoftheNorthAmericancontinent.
A.Seldommorethan40feetwideand12feetdeep,butitran363milesacrossthe
ruggedwildernessofupstateNewYork,theErieCanalconnected
"Seldommorethan40feetwideand12feetdeep,butitran363milesacrossthe
ruggedwildernessofupstateNewYork"isamodifierandithas''but''.
after''but'',wedohaveaclause
butnotbefore(Seldommorethan40feetwideand12feetdeep).so||ismmissing.
B.Seldommorethan40feetwideor12feetdeepbutrunning363milesacrossthe
ruggedwildernessofupstateNewYork,theErieCanalconnected
C.Itwasseldommorethan40feetwideand12feetdeep,andran363milesacross
theruggedwildernessofupstateNewYork,buttheErieCanal,connecting
D.TheErieCanalwasseldommorethan40feetwideor12feetdeepanditran363
milesacrosstheruggedwildernessofupstateNewYork,whichconnected
E.TheErieCanal,seldommorethan40feetwideand12feetdeep,butrunning363
milesacrosstheruggedwildernessofupstateNewYork,connecting
''and'',''but''areinterchangedinthisquestion.bothhasdifferentmeaning.

Logically,thesentencecontainsthecontrast.eliminateC,D
.

TheSupremeCourthasruledthatpublicuniversitiescancollectstudentactivityfeesevenwithstudents'objections
toparticularactivities,solongasthegroupstheygivemoneytowillbechosenwithoutregardtotheirviews.
(A)withstudents'objectionstoparticularactivities,aslongasthegroupstheygivemoneytowillbe
(B)iftheyhaveobjectionstoparticularactivitiesandthegroupsthataregiventhemoneyare
(C)iftheyobjecttoparticularactivities,butthegroupsthatthemoneyisgiventohavetobe
(D)fromstudentswhoobjecttoparticularactitivitiessolongasthegroupsgivenmoneyare
(E)thoughstudentshaveanobjectiontoparticularactivities,butthegroupsthataregiventhemoneybe
''eventhough''and''evenif''donotconveythesamemeaning.
forex:
shelikestosingeventhoughsheisnotverygood.
theactionafter''eventhough''istrueandalwaysapplies.
shestudieseverynightevenifshe'stired.
theactionafter''evenif''onlyhappenssometimes.
InthegivensentenceE,itisillogicaltoimplythatallstudentswilldefinitelyobjecttoanactivity.

Whendriveinswereattheheightoftheirpopularityinthelate1950's,some4,000existedintheUnitedStates,but
todaytherearelessthanonequarterthatmany.
A.therearelessthanonequarterthatmany
B.therearefewerthanonequarterasmany
C.therearefewerthanonequarterofthatamount
D.thenumberislessthanonequartertheamount
E.itislessthanonequarterofthatamount
B.therearefewerthanonequarterasmany

B should be the correct answer. In this case last part ( as there were) is implicit as
this answer choice already has the same structure ( there are).

------------------------------------------------------------------''because of'' vs because:


It is cold in Arizona because of the westward storm coming from pacific.
This sentence is correct because it presents the reason why it is cold in Arizona. It is is
cold there because of the westward wind. Now, the verb-ing modifier presents extra
information about this westward wind. This modifier specifies its direction. But, the core

meaning of the sentence is that it is cold in Arizona because of the westward storm.
In the same line, let;s analyze the problem sentence that you have presented.
Because of Elnath Industries posting a second consecutive quarter of losses, its stocks
tumbled 20% in the last three days.
The core meaning of the sentence is that Elnath Industries' stocks fell 20% in the last
three days because of Elnath Industries. Now, the verb-ing modifier "posting..." is
providing extra information about this industry. So if we exclude this information, the
core meaning conveyed is illogical. But, this is not the case with the Arizona sentence.
Hence, the correct way to convey the logical meaning is to say: Because Elnath
Industries posted second consecutive quarter of losses, its stocks tumbled 20% in the
last three days.
This sentence correctly presents the reason why the industry's stocks fell so much.
----------------------------correct const:
x causes the same as Y
It causes the same to run X as for Y
Incorrect const:
x causes the same as for Y
It causes the same to run X as Y( for removed)
------------------------------GMAT Prep Question 1
Industrialization and modern methods of insect control have improved the standard of living around the globe while at the
same time they have introduced some 100,000 dangerous chemical pollutants, having gone virtually unregulated
since they were developed more than 50 years ago.
A. while at the same time they have introduced some 100,000 dangerous chemical pollutants, having ---> I selected this
choice
B. while at the same time introducing some 100,000 dangerous chemical pollutants that have ---> correct choice
C. while they have introduced some 100,000 dangerous chemical pollutants at the same time, which have
D. but introducing some 100,000 dangerous chemical pollutants at the same time that have
E. but at the same time introducing some 100,000 dangerous chemical pollutant, having
"while"

is a subordinating construction, not a coordinating one, so it doesn't have the same strict parallelism requirements as

does "but".
"while" can be followed by an -ing participle. (it can also be followed by an adjective or adjective phrase.)
"while" is NOT a parallel construction marker.
E.g., I normally write letters while driving on the freeway.

GMAT Prep Question 2


Diesel engines burn as much as 30% less fuel than gasoline engines of comparable size, as well as emitting far less
carbon dioxide gas and far fewer of the other gasses that have been implicated in global warming.
A of comparable size , as well as emitting far less carbon dioxide gas and far fewer of the other gasses that have --> correct
choice
B of comparable size, as well as emit far less carbon dioxide gas and far fewer of the other gasses having --> I selected this
choice
C of comparable size, and also they emit far fewer carbon dioxide and other gasses that have
D that have a comparable size, and also they emit far fewer of the other gasses having
E that have a comparable size, as well as emitting far fewer of the other gasses having

---------------------------

Today, because of improvements in agricultural technology, the same amount of acreage


produces double the apples that it has in 1910.

(A) double the apples that it has


(B) twice as many apples as it did
(C) as much as twice the apples it has
(D) two times as many apples as there were
(E) a doubling of the apples that it did
'double the apples' is awkward.
(B) twice as many apples as it did- correct
C) as much as twice the apples it has
much- used for uncountable
(D) two times as many apples as there were (two times- wordy)
(E) a doubling of the apples that it did
a doubling of the apples' is awkward.

----------------If you say "twice as many", then this construction should be paired with a countable
noun.
e.g., twice as many dogs --> "dogs" is a countable noun

If you said "twice as much", then this construction should be paired with an
uncountable noun.

e.g., twice as much water --> "water" is an uncountable noun

If the noun in question is already an explicitly numerical quantity, then you should
use neither "much" nor "many". Instead, you should just use "twice" or "double" by
itself.
e.g., twice the increase --> "increase" is an explicitly numerical quantity

These rules are followed pretty closely.


For instance:
Twice as much water --> correct, since "water" is an uncountable noun (but is not an
explicitly numerical quantity)
Twice the increase... --> correct
Twice the water... --> incorrect, since water is not a numerical quantity
Twice as much as the increase... --> incorrect; redundant
---------------while" MUST be used with a clause or __ing phrase. it can't be used with a
construction that doesn't contain any sort of verb form (such as this one, which is
just noun + prepositional phrase)
----------------------------------------------------------------Conjunctive Adverbs
These words sometimes act as conjunctions and at other times, as adverbs accordingly, in fact, again, instead, also,
likewise, besides, moreover, consequently, namely, finally, nevertheless, for example, otherwise, further, still,
furthermore, that is, hence, then, however, therefore, indeed, thus
When two independent clauses are joined by a conjunctive adverb we need to insert a semicolon between the two
clauses. Note that conjunctive adverbs are not really full conjunctions, and they cant do that job by themselves. It is the
semicolon that does the real job of joining the two independent clauses.
Examples:
The rain slashed the town; furthermore, the people scurried for shelter.
The policeman dodged the bullets; however, a bystander was shot.
Note that if we use a comma instead of a semicolon in the examples above, we will create a comma splice.
------------------------------that/those makes a new copy, it does not.
--------------------Archaeologists in Ireland believe that a recently discovered chalice, which dates from the eighth century, was probably
buried to keep from being stolen by invaders.
(A) to keep from

(B) to keep it from


(C) to avoid
(D) in order that it would avoid
(E) in order to keep from
Ask yourself who buried the chalice and why. They buried it "to keep it from being stolen", not "to keep from being stolen

------------------------------------------------------------Conjunctions are the words that join together sentences and words. There are following types of conjunctions:

Coordinating Conjunctions:

These conjunctions join together nouns, pronouns, verbs, adjectives, clauses, and propositional phrases. These
conjunctions are For, And, Nor, But, Or, Yet, So. In short, they are called FANBOYS.
Mary wanted to perform tonight, but her health did not permit her. (joining two independent clauses)
Samuel went to Paris to visit the tourist places and to write his thesis. (joining two to verb phrases.

Subordinating Conjunctions:
These conjunctions join a clause to another to communicate the full meaning of the sentence. They may provide a
necessary transition between the two ideas in two clauses in the sentence. This transition can indicate time, place or cause
and effect relationship.

The teacher will announce the date of assessment once all the students submit their projects.
John watched his favorite TV show after he finished his project.
Since Amy was getting late for the concert, she asked her friends to meet her directly at the venue.
They may also reduce the importance of one clause to make the more important idea of the two obvious. The more
important idea lies in the main clause, while the less important idea lies in the clause followed by the subordinating
conjunction.

Although the day is pleasant, it does not look apt for picnic.
As he saw his father approaching, Joe hung up the phone.

Correlative Conjunctions:
Some conjunctions are used in pairs. They connect two equal grammatical entities. These conjunctions are EitherOr,
NeitherNor, Not OnlyBut Also. BothAnd, etc. Always make sure that the entities following the two conjunctions must
be grammatically and logically parallel to each other.

Amy not only baked cake but also cooked sumptuous meal.
Entities parallel Verb phrases = baked cake and cooked sumptuous meal.

The teacher does not care whether you stay or go.


Entities parallel Verbs = stay and go

---------------------------------------------------------A patient accusing a doctor of malpractice will find it difficult to prove damage if there is a lack of some
other doctor to testify about proper medical procedures.
(A) if there is a lack of some other doctor to testify
(B) unless there will be another doctor to testify
(C) without another doctors testimony
(D) should there be no testimony from some other doctor
(E) lacking another doctor to testify

I believe that " a lack of" is not wrong.


Owing to a lack of supporters, the reforms did not succeed.
http://www.thefreedictionary.com/lack
A is not completely wrong but there are few issues:
1) There are few cats in my house./ Few cats are in my house.
What's the difference in these two sentences? There must be a subtle difference otherwise two
different structures would not have existed.
In first case, I want to emphasize the existence of cats. (their location is an afterthought)
In second case, We already know that there are cats, the point is to specify their location.
What is important here? A testimony about a proper medical procedure ( can be provided
anybody..doesn't matter...It could be doctor or Research associate or... who knows the right medical
procedure.) OR Lack of doctor who can testify.
It is clear that testimony has more importance here as it will prove malpractice. So the focus should be
on that.
2) Some other Vs Another
They almost have the same meaning. But there is a small difference.

If you give me another chance, I will crack the GMAT. (Right...also sounds right)
If you give me some other chance, I will crack the GMAT. (Wrong...does not sound right)

The difference is that another is specific/ More definite.


Forbes Magazine sentence:
When you toss responsibility for that news to some other leader, the lesson gets lost. (Correct)
Some other is used when you want to be vague/Not specific.
It is better to use here "Another" as it conveys the better meaning though some other is not completely
wrong here.
B) Option: In logic "unless = If NOT"
If I don't get around 760 in my mock test, I will not take GMAT.
Unless I get around 760 in my mock test, I will not take GMAT.
Unless, Without, If not....They are very similar in terms of Logical structure. It also helps in CR to solve
the questions.
D) Use of should in conditional:
Should you fail to get good score in GMAT, you can always take it.

Here I don't think you will fail but in case you fail you can retake it.
Should implies nonsensical meaning in option D.
E) Lacking modifies damage which is nonsensical.
-----------------------------------------------------------------------------------Wisconsin, Illinois, Florida, and Minnesota have begun to enforce statewide bans prohibiting landfills
to accept leaves, brush, and grass clippings.
(A) Same
(B) prohibiting that landfills accept leaves, brush, and grass clippings
(C) prohibiting landfills from accepting leaves, brush, and grass clippings
(D) that leaves, brush, and grass clippings cannot be accepted in landfills
(E) that landfills cannot accept leaves, brush, and grass clippings
To use the word 'ban' correctly, you have to say that the ban prevents (or prohibits, or proscribes, or
criminalizes, or one of the many other words carrying the meaning of 'puts off limits') something. You
can't say 'a ban that X does Y'. So that does it for the last two choices.
As an alternative construction - possibly the most common construction, actually - you can say a ban
ON (NOUN). This construction could theoretically be used here, but (1) it isn't, and (2) it would be
difficult to introduce it into this particular sentence without some seriously awkward phrasing.
A and B are done in by poor idiomatic usage. As far as A, you can't say 'X is prohibited TO (verb)',
although you can say the related 'It is prohibited to (verb)'. You have to say that X is prohibited FROM
(verb)ing'. And B should look really, really wrong to just about any native writer/speaker of English.
---------------------------------------------Under high pressure and intense heat, graphite, the
most stable form of pure carbon, changes into the
substance commonly referred to as diamond and
remaining this way whether or not the heatand
pressure are removed.
(A) remaining this way whether or not
(8) remaining like that even as
(C) remaining as such whether or not
(D) remains in this way although
(E) remains thus even when
A,B, C can be eliminated for obvious reasons.
left with D and E.
the usage of although is wrong in D. as it is only used in case of contrast. here, there is no contrast.
-------------------------------------

A recent study has found that within the past few years, many doctors had elected early retirement
rather than face the threats of lawsuits and the rising costs of malpractice insurance.
(A) had elected early retirement rather than face
consider this:
I have elected John rather than Abhay.
'' elected early retirement'' seems like that they have elected " early retirement".
(B) had elected early retirement instead of facing
(C) have elected retiring early instead of facing
same as A.
(D) have elected to retire early rather than facing
(E) have elected to retire early rather than face
---------------------A study commissioned by the Department of Agriculture showed that if calves exercise and associated
with other calves, they will require less medication and gain weight quicker than do those raised in
confinement.
(A) associated with other calves, they will require less medication and gain weight quicker than do
(B) associated with other calves, they require less medication and gain weight quicker than
(C) associate with other calves, they required less medication and will gain weight quicker than do
(D) associate with other calves, they have required less medication and will gain weight more quickly
than do
(E) associate with other calves, they require less medication and gain weight more quickly than
quicker/ more quickly:
quicker is an adjective while '' more quickly'' is an adverb.and an adj cannot be used to modify a
verb in this case ''gain''.
eliminate A,B.C.

---------------------------------------------------------------------------

The rise in the Commerce Department's index of leading economic indicators suggest that the
economy should continue its expansion into the coming months, but that the mixed performance of
the index's individual components indicates that economic growth will proceed at a more moderate
pace than in the first quarter of this year.
A) suggest that the economy should continue its expansion into the coming months, but that
B) suggest that the economy is to continue expansion in the coming months, but
C) suggests that the economy will continue its expanding in the coming months, but that
D) suggests that the economy is continuing to expand into the coming months, but that
E) suggests that the economy will continue to expand in the coming months, but
please note that the usage of '''that'' in options A,C, and D is wrong.it makes 2 sent. it changes the
meaning of sent. by making 2 sent. ||el that should not be.

-----------------------------

According to some economists, the July decrease in unemployment so that it was the lowest in two
years suggests that the gradual improvement in the job market is continuing.
(A) so that it was the lowest in two years
(B) so that it was the lowest two-year rate
(C) to what would be the lowest in two years
(D) to a two-year low level
(E) to the lowest level in two years
"what" is one of those words that should be evaluated on a case-by-case basis. generally, you shouldn't
use it unless there are no other options. (the good thing here, of course, is that these questions are
multiple-choice -- so you'll be able to tell whether there are "other options" or not!
(D) to a two-year low level
the principal objection here is that the meaning is incorrect/unclear: for instance, a 'two-year low
level' could mean that it hit a low level and then stayed there for two years.
consider other uses of the adjective "two-year":
a two-year lease is a lease that LASTS for two years.
a two-year recession is a recession that LASTS for two years.
a two-year marriage ... etc. etc.
by analogy, we can see that this answer choice has (or at least primarily suggests) the incorrect
meaning: a two-year low level would be a low level that lasted for two years. that's not the intended
meaning in the problem.
(E) to the lowest level in two years
--------------------------------------

with the aim to- wrong idiom


with the aim of- correct
----------------------------In recent years cattle breeders have increasingly used crossbreeding, in part that their steers should
acquire certain characteristics and partly because crossbreeding is said to provide hybrid vigor.
(A) in part that their steers should acquire certain characteristics
(B) in part for the acquisition of certain characteristics in their steers
(C) partly because of their steers acquiring certain characteristics
(D) partly because certain characteristics should be acquired by their steers

(E) partly to acquire certain characteristics in their steers- correct


------------------------

Declining values for farm equipment and land, the collateral against which farmers borrow to get
through the harvest season, is going to force many lenders to tighten or deny credit this spring.

(A) the collateral against which farmers borrow to get through the harvest season, is
(B) which farmers use as collateral to borrow against to get through the harvest season, is
(C) the collateral which is borrowed against by farmers to get through the harvest season, is
(D) which farmers use as collateral to borrow against to get through the harvest season, are
against is misplaced here. it should be placed next to collateral.
(E) the collateral against which farmers borrow to get through the harvest season, are
-----------------------------------------

Except for a concert performance that the composer himself staged in 1911Scott Joplin's ragtime
operaTreemonishawas not produceduntil 1972sixty-one years after its completion
(A) Except for a concert performance that the composer himself staged - correct
(B) Except for a concert performance with the composer himself staging it
"a performance with the composer staging..." implies that the composer was actually staging stuff during the performance. Nonsense, that.

(C) Besides a concert performance being staged by the composer himself


usage of ''being'' wrong.
(D) Excepting a concert performance that the composer himself staged
wrong usage of excepting.
(E) With the exception of a concert performance with the staging done by the composer himself.
same as B.
AWKWARD AND wordy.
---------------------------* it's unidiomatic to use "allow" with the subjunctive.
the correct usage is "allow NOUN to be ..."
it's incorrect to say "allow that NOUN be ..."
-----------------------the only time i've seen it used appropriately is to say something like this:
"having been/done X, [independent clause here]"
this indicates that after completing the first part, the subject does whatever it is doing in the
independent clause..
-------------------------------------------------------

Much can be used as an adverb modifying verb in a sentence,


For ex:

However much United States voters may agree that there is a waste in government..............
Many is always an adjective.
--------------------------------------------

** in some situations, you do have to repeat the subordinator, but not for the sake of parallelism -rather, for the sake of getting rid of ambiguity.
for instance:
claire said that she finished the test early but her sister took the whole period.
--> AMBIGUOUS
this could mean
(a) claire said both of these things [claire said (X and Y)]
, or (b) claire said the first thing but the narrator knows the second one firsthand [(claire said X)
and Y)]
claire said that she finished the test early but that her sister took the whole period.
--> UNAMBIGUOUS
now we know for sure that claire said both of these things
-----------------------------

In no other historical sighting did Halleys Comet cause such a worldwide sensation as did its return in
1910-1911.
(A) did its return in 19101911
(B) had its 19101911 return
(C) in its return of 19101911
(D) its return of 19101911 did
(E) its return in 19101911
it's actually ambiguous. the ambiguity is hard to see, especially because this choice comes last (so that
the correct meaning has already been hammered into your head by the other answer choices), so i'll
use an analogy to explain.
in no other music festival did jimi hendrix cause such a spectacle as woodstock.
this is ambiguous. there are two possible interpretations:
(1) ...as IN woodstock (this is analogous to the intended meaning of the posted problem)
(2) ...as DID woodstock (i.e., you're comparing the spectacle caused by jimi hendrix to the spectacle
caused by woodstock itself)
the same two types of interpretations are possible for the posted problem.
-here's a relevant takeaway:
if you have 2 ANSWER CHOICES THAT DIFFER ONLY IN THE PRESENCE/ABSENCE OF A LITTLE WORD, then
CHANCES ARE THAT THE LITTLE WORD IS NECESSARY.
this is a really, really, really strong bias, like over 90% likely. if you see two such choices, then you
should REALLY think twice before picking the one without the "little word"

i can't guarantee this 100%, but LOOK FOR AN AMBIGUITY that arises if the little word is omitted.
* if the little word isn't the _sole_ difference between 2 answer choices, then try to concentrate on the
other differences between the answer choices first.
----------------------------------

Whereas in mammals the tiny tubes that convey nutrients to bone cells are arrayed in parallel lines, in
birds the tubes form a random pattern.
(A) Whereas in mammals the tiny tubes that convey nutrients to bone cells are arrayed in parallel lines,
in birds the tubes
(B) Whereas the tiny tubes for the conveying of nutrients to bone cells are arrayed in mammals in
parallel lines, birds have tubes that
Here, ''whereas'' is used to compare 'the tiny tubes' with 'birds' ,which is wrong.
(C) Unlike mammals, where the tiny tubes for conveying nutrients to bone cells are arrayed in parallel
lines, birds tubes
(D) Unlike mammals, in whom the tiny tubes that convey nutrients to bone cells are arrayed in parallel
lines, the tubes in birds
(E) Unlike the tiny tubes that convey nutrients to bone cells, which in mammals are arrayed in parallel
lines, in birds the tubes
as per Ron about ans choice E,
such indirect comparison are never seen to be correct in GMAT.( the tiny tubes, in birds the tubes)
For the comparison to work correctly, "the tubes" should begin that clause. It's not acceptable for "in birds" to intervene.

--------------------------------------------------Although the term psychopath is popularly applied to an especially brutal criminal, in psychology it is
someone who isapparently incapable of feeling compassion or the pangs of conscience.
(A) it is someone who is
(B) it is a person
(C) they are people who are
(D) it refers to someone who is - correct
(E) it is in reference to people
apart from obvious errors,
will talk only about E":
In e, it refers to ''the term psychopath'' which is singular while ''people '' is plural.hence,wrong.
refer vs reference- VAN
----------------------------------------------

Usage of 'whereas'':
"Whereas" is a subordinating conjunction: it must be placed before a whole clause, which, in turn, must
be connected to another whole clause. (i.e., the words that follow whereas must be a complete
sentence by themselves, and those words must be attached to another complete sentence.)
Example : Whereas Mary likes cookies, John prefers ice cream.
-------------------------------Cajuns speak a dialect brought to southern Louisiana by the 4,000 Acadians who migrated there in
1755; their language is basically seventeenth-century French to which has been added English,
Spanish, and Italian words.
(A) to which has been added English, Spanish, and Italian words
(B) added to which is English, Spanish, and Italian words
(C) to which English, Spanish, and Italian words have
been added
(D) with English, Spanish, and Italian words having been added to it
(E) and, in addition, English, Spanish, and Italian words are added
apart from obvious errors,
have been added / are added:
''are added'' implies that this action has been done just now while ''have been added'' clearly
implies that action was done in the past.

------------------------

Australian embryologists have found evidence that suggests that the elephant is descended from an
aquatic animal, and its trunk originally evolved as a kind of snorkel.
A. that suggests that the elephant is descended from an aquatic animal, and its trunk originally evolved
B. that has suggested the elephant descended from an aquatic animal, its trunk originally evolving
C. suggesting that the elephant had descended from an aquatic animal with its trunk originally evolving
D. to suggest that the elephant has descended from an aquatic animal and its trunk originally evolved
E. to suggest that the elephant is descended from an aquatic animal and that its trunk originally
evolved
apart form obvious errors:

elephant has descended: active voice.


elephant is descended: passive voice
1st one is like elephant are the doer.awkward.
"has descended" means "has moved downward". this can be in either a literal sense (he has descended to sea level from a height of 8000 feet) or a
metaphorical sense (i don't want to descend to the level of common street thugs), but it can't refer to ancestry.

----------------------------------------------------

Joan of Arc, a young Frenchwoman who claimed to be divinely inspired, turned the tide of English victories in her
country by liberating the city of Orleans and she persuaded Charles VII of France to claim his throne.
(A) she persuaded Charles VII of France to claim his throne
(B) persuaded Charles VII of France in claiming his throne
(C) persuading that the throne be claimed by Charles VII of France
(D) persuaded Charles VII of France to claim his throne
(E) persuading that Charles VII of France should claim the throne
correct idiom: persuade X to verb

------------------------------------------------------------------------The use of lie detectors is based on the assumption that lying produces emotional reactions in an
individual that, in turn, create unconscious physiological response.
A) that, in turn, create unconscious physiological responses.
B) that create unconscious physiological responses in turn.
C) creating, in turn, unconscious physiological responses.
D) to create, in turn, physiological responses that are unconscious.
E) who creates unconscious physiological responses in turn
x causes y and y, in turn, causes z.
here, ''in turn'' is used to show that y causes z. if we move ''in turn'' at the end of sent., it changes
the meaning by making it ambiguous.
--------------------------------------------------------------------------------------------------------------

Nevertheless is strictly an adverb. You can't use it to connect two sentences.


-------------------------------------------------------------

he was less successful after she had emigrated to New York compared to her native
Germany, photographer Lotte Jacobi nevertheless earned a small group of discerning admirers, and her
photographs were eventually exhibited in prestigious galleries across the United States.
A) She was less successful after she had emigrated to New York compared to
incorrect comparison.
B) Being less successful after she had emigrated to New York as compared to
C) Less successful after she emigrated to New York than she had been in- correct
this sort of modifier -- an initial modifier that begins with an adjective -- should fulfill the following two requirements:
(1) the adjective should refer to the subject that follows the comma;
(2) the description in the modifier should have some sort of easily understandable relationship to whatever is described in the
following clause

D) Although she was less successful after emigrating to New York when compared to

E) She had been less successful after emigrating to New York than in
Analysis:
A, E are run on sentences.connected by comma.
less.....compared to- wrong
less....than-

correct

yet another awesome explanations by Ron:


if you have a sentence that says "compared to/with" (or similar constructions, including "in comparison
to", "as compared to/with", etc.), then the sentence CANNOT also use a comparison word, such
as more, less, greater, prefer, better, worse, X times as much, etc.
to use "compared to/with", you just STATE statistics, without using any other comparison word.
examples:
this year's unemployment rate of 12% is three times as great compared to the rate in 1994 -->
incorrect (redundant)
the correct way to write it:
this year's unemployment rate is 12%, compared to 4% in 1994
i.e., you just come out and SAY the statistics, and then give "compared to".

---------------------------------------------------------

By Ron:
Usage of "being":
the short answer to this question: basically, "being" is ok when there are no acceptable alternative
formulations that DON'T use it.
from what we've seen, "being" can be ok as long as at least one of the following 2 conditions obtains:
(1) it is part of a PASSIVE-VOICE construction (note that such constructions require a form of "to be", so,
if the passive voice is in the -ing form, that form will manifest as "being");
or
(2) it is used as a ING noun (i.e., the action of "being something" is treated as a NOUN in the
sentence).
most, but not all, of these nouns form of a construction in the passive voice, as discussed in #1 above.
examples: (the first one i made up; the other three are correct answers to official problems)
Being followed by paparazzi 24 hours a day has caused many celebrities to become extremely hostile
to strangers. (source: i made this up on the spot)
(note that this is both --> it's a passive-voice ING noun!)
During the 1950s, as part of their therapy, young polio victims learning to live with their disabilities
were helped to practice falling, so that they could learn to fall without being hurt.(source: gmat

prep)
(again, passive voice ING noun)
According to one expert, the cause of genetic irregularities in many breeds of dog is not so much that
dogs are being bred for looks or to meet other narrow criteria as that the breeds have relatively few
founding members. (source: gmat prep)
(passive; not ING noun)
Being heavily committed to a course of action, especially one that has worked well in the past, is
likely to make an executive miss signs of incipient trouble or misinterpret them when they do
appear. (source: gmat prep; also in the OG verbal supplement)
(ING noun; not passive)
-------------------------------------------------------------------------------------------------------------------

A firm that specializes in the analysis of handwriting claims from a one-page writing sample that it
can assess more than three hundred personality traits, including enthusiasm, imagination, and
ambition.
(A) from a one-page writing sample that it can assess
seems that the firm claims from a one-page writing sample.illogocal.
(B) from a one-page writing sample it has the ability of assessing
(C) the ability, from a one-page writing sample, of assessing
(D) to be able, from a one-page writing sample, to assess
(E) being able to assess, from a one-page writing sample,
correct construction:
claims + that
claims + to
-------------------------------------------------------

Between 1990 and 2000 the global economy grew more than it did during the 10,000 years
from the beginning of agriculture to 1950.
A) Between 1990 and 2000 the global economy grew more than it did during the 10,000 years from the
beginning of agriculture
(B) Between 1990 and 2000 the growth of the global economy was more than that during 10,000 years,
from when agriculture began.
You cannot use that in this sort of construction, because constructions using that of must
have EXACTLY parallel structures. If the second half says that during 1000 years, then the
preceding half must say the growth of ___during something else (or some other time
preposition, such as before or after, in place of during) -- Taken from Ron
Note that there is nothing wrong with 'from when agriculture began'
(C) The growth of the global economy between 1990 and 2000 exceeds that which had been for 10,000
years from the beginning of agriculture

exceeds is incorrect
usage of past perfect is 100% incorrect
usage of that which is awkwar
(D)The growth of the global economy between 1990 and 2000 exceeds what it has been for 10,000
years, from when agriculture began
usage of has been is incorrect here.
(E) The growth of the global economy between 1990 and 2000 exceeded what it did for the 10,000
years from the beginning of agriculture
'what it did' does not make sense. The growth didn't do anything
------------------------------------A mutual fund having billions of dollars in assets will typically invest that money in hundreds
of companies, rarely holding more than one percent of the shares of any particular corporation.
(A) companies, rarely holding more than one percent - CORRECT
(B) companies, and it is rare to hold at least one percent or more(C) companies and rarely do they hold more than one percent
(D) companies, so that they rarely hold more than one percent
(E) companies; rarely do they hold one percent or more
B,C- usage of nd' wrong. it changes the meaning. original meaning- cause and effect
In B, 'it' should refer to mutual fund but it is used as a dummy pronoun.
for ex: it is raining.
-------------------------------George Sand (Aurore Lucile Dupin) was one of the first European writers to consider the rural poor to
be legitimate subjects for literature and portray these with sympathy and respect in her novels.
(A) to be legitimate subjects for literature and portray these
(B) should be legitimate subjects for literature and portray these
(C) as being legitimate subjects for literature and portraying them
(D) as if they were legitimate subjects for literature and portray them
the usage of s if' is wrong in this.it changes the meaning. seems that rural poor were not actually
''legitimate subjects for literature''
(E) legitimate subjects for literature and to portray them
-------------------------------The world wildlife fund has declared that global warming, a phenomenon most scientsts agree to be
caused by human beings in burning fossil fuels, will create havoc among migratory birds by altering
the environment in ways harmful to their habitats.

a) A phenomenon most scientists agree to be caused by human beings in burning fossil fuels, because of the usage of 'to be ', it seems like the cause is not yet occured.
b) a phenomenon most scientists agree that is caused by fossil fuels burned by human beings,
cause is not the 'fossile fuels'itself, but ''the burning of fossil fuels ''.
c) a phenomenon that most scientists agree is caused by human beings' burning of fossil fuels
d) which most scientists agree on as phenomenon caused by human beings who burn fossil fuels, changes the meaning
e) which most scientists agree to be a phenomenon caused by fossil fuels burned by human beings
'to be'is generally used by GMAT to make a sent. wordy.
------------------------------------"the only words that can toggle between singular and plural are
(i) collective nouns (like 'team', 'band', or 'faculty'), which can be singular or plural depending on the
context,
(ii) words whose singular and plural forms are the same (like 'moose' or 'fish'). "
-Ron
---------------------------------Today's techonology allows manufacturers to make small cars more fuel efficient now than at any time in
their production history.
a. Same as above
b. small cars that are more fuel-efficient than they were at any time in their
c. small cars that are more fuel-efficient than those at any other time in
d. more fuel-efficient small cars than those at any other time in their
e. more fuel-efficient small cars now than at any time in

the sentence is supposed to say that today's small cars are more fuel-efficient than previous small cars -i.e., OTHER small cars.
this is important, because you can't use a "they"/"their" construction (which would illogically imply that
you're talking about thesame small cars mentioned in the first part of the sentence).
the use of "those" in (c), on the other hand, accomplishes this distinction nicely.
analogy:
sprinters in texas can run faster than they can in north dakota.
--> illogical; in this sentence, "they" would be taken to mean "sprinters in texas" (i.e., not just sprinters).
sprinters in texas can run faster than those in north dakota.
--> makes sense; "those" = sprinters, in this case.
same thing with "they/their" vs. "those" in these instances.

Good work identifying the pronoun error. To clarify, however, it is not an error in number (it v. their) but
an error in ambiguity. In this case, "their" could refer either to "manufacturers" or "cars"; thus, we can
eliminate any answer choices that include "their."
I also agree that the primary error in (e) is redundancy - Because the sentence is discussing "today's"
technology it is redundant to say "now." I also think that the meaning of (e) is ambiguous: are
manufacturers making cars that are more fuel efficient, or are manufacturers simply making more cars
(number of cars)?
if a noun is preceded by adjectives, then any pronoun referring to that noun MUST refer to the complete
package of adjectives + noun.
so, in choice (d), "their" must refer to "fuel-efficient small cars"; that doesn't make sense, since fuelefficient small cars have not been made throughout the entire production history in question.

This model explains all known subatomic particles, some of them were only recently
discovered.
---------------------------------------------------------------

the dashes are basically equivalent to commas. they are used in one of three situations:
1) EMPHASIS
the above sentence is an example of this usage; the idea is that the figure (over 2000 jobs) is strikingly
high, so the description is set off with dashes rather than commas to emphasize it.
2) IRONY
note the following two sentences:
john, who is from colombia, thinks that california winters are too cold.
vs.
tim -- who is from alaska -- thinks that california winters are too cold.
the first sentence makes perfect sense, in terms of common sense -- colombia is hot all the time, so it's
reasonable for somebody who is from there to think that california is cold.
the second sentence doesn't really make any sense; it's ironic that somebody from alaska (which is much,
much colder than california) would find california to be too cold. therefore, because of the irony in this
modifier, the modifier is set off by dashes rather than by commas.
3) WHEN COMMAS WOULD CREATE AMBIGUITY
e.g.
in the picture were three of my friends, john, tim, and bob.
this sentence is ambiguous -- it could refer to three of my friends (whose names are john, tim, and bob),
or it could refer to six different people (three of my friends and also john, tim, and bob).
in the picture were three of my friends -- john, tim, and bob it.
not ambiguous; there are definitely only three people under discussion.
---------------------------------------------

Because of wireless service costs plummeting in the last year, and as


mobile phones are increasingly common, many people now using their mobile
phones to make calls across a wide region at night and on weekends, when
numerous wireless companies provide unlimited airtime for a relatively small
monthly fee.

A. Because of wireless service costs plummeting in the last year, and as mobile
phones are increasingly common, many people
B. As the cost of wireless service plummeted in the last year and as mobile phones
became increasingly common, many people
C. In the last year, with the cost of wireless service plummeting, and mobile phones
have become increasingly common, there are many people
D. With the cost of wireless service plummeting in the last year and mobile phones
becoming increasingly common, many people are - OA
E. While the cost of wireless service has plummeted in the last year and mobile
phones are increasingly common, many people are
This is what Ron has to say here

choice (e) can be knocked out for its bad parallelism.


the two actions described at the beginning - cost plummeting, phones becoming more common - are clearly simultaneous, so we need to use parallel
structure for these simultaneous events.
the first is "the cost ... has plummeted", so the second MUST be "phones ... have become increasingly common". it's not written that way, so (e) is
incorrect.
in addition, (e) uses while in a questionable sense. when while appears at the beginning of a clause like this, it's normally used to
introduce contrast ("while most of the class spent the weekend studying for class, i spent the weekend at the beach").
-now, an analysis of the grammatical weirdness that's going on in the correct answer:
i'm learning something new here, but that happens decently often with these things. sentence correction is a slippery fish.
it appears that 'with' may be used with an __ING form to represent circumstances that are contemporaneous with the action described
in the main clause.
that's what's going on here: the "with ___ing" constructions are used to describe things that are contemporaneous with the described uptick in the use
of mobile phones.
"in the last year" = within the last 12 months. this will almost exclusively be used with "has VERBed".
"last year" = sometime last year (i.e., in the past). this will be used with past tenses (VERBed, had VERBed).

=------------------------------"With surface temperatures estimated at minus 230 degrees Farenheit, Jupiter's


moon Europa has long been considered far too cold to support life, and with 60
square miles of water though to be frozen from top to bottom."
[4:05:06 PM] Rajat Gugnani: "At" is used to refer a specific location. Here when you
say the temperature is estimated at minus 230 degree fahrenheit, you are referring
to that location. One of the main usage of "at" is to refer a specific location.
---------------------------------Usage of 'and also':
Also - The meaning of also is "in addition to"/ "besides". It is like when you are
giving some additional information about something.

Selma Lagerlf became the first woman and also the first Swedish writer to win the
the Nobel Prize for Literature.
Here these two things dont have equal value. Of course becoming the first woman
has a lot more importance than becoming the first Swedish writer. Therefore we can
use "and also"/ "in addition to" / "besides" here.

Selma Lagerlf became the first woman in addition to the first Swedish writer to win
the the Nobel Prize for Literature. (correct)

Selma Lagerlf became the first Swedish writer in addition to the first woman to win
the the Nobel Prize for Literature. (Incorrect)

OG explanation is shitty because then "and also" will always be wrong.

Here in this question "Einstein's theory of relativity" and "the better ability to
observe extremely distant astronomical objects." these are the two things.

Here they both have equal importance. the second is not providing any additional
information. therefore, using "both X and Y will be a perfect structure because this
structure makes it very clear that they both had equal importance.
-------------------------such....that
technically you used the such X that Y construction, which is perfectly fine. This is
not the same as the direct construction of "such that".

In addition, the proper way to phrase that sentence would be "The king fought with
such bravery and passion that..." such is followed by a noun.
-------------------

In such X, X is a noun. In so X, X is an adjective:

Her eyes are so beautiful that I am hypnotized by them.


Her eyes are of such beauty that I am hypnotized by them.
-----------------------------------

So that and such that -> provides reasons


--------------There are clauses of consequence that appear regularly on the GMAT:
1. so that

The Berlin Airlift provided the city with food and valuable resources, so that the
vitally important city would not fall into Soviet control.
2. so as to

Upon his election, Pope John Paul II decided to learn Spanish, so as to communicate
directly with the single largest group of Catholics in the world.
3. such that

The local rulers of early medieval Europe were almost constantly at war, such that
the infrastructure had little chance to develop beyond the most rudimentary level.
4. so [adjective] that
5. so [adjective] as to
6. such [noun] that
In all six of these, what precedes these words it the causing situation or condition, and what follows them is what results as a consequence. This is a standard
construction for demonstrating the purpose of an action. The four with that require a full [noun] + [verb] clause, and the two with to demand
an

infinitive phrase.

The other three are really important (#4,5, and 6)


In the two structures that involve an adjective (#4,#5), it must be an adjective that admits of degree i.e. one could be more or less of this adjective. One can be
more happy or less happy, more tired or less tired, more rich, more healthy, more educated, etc.

Some adjectives have an all-or-nothing quality left-handed, electric, financial, individual, etc. one cannot be more or less or any of these. Technically, the
word unique is in this latter category something is either unique or it isnt, and it is logically incorrect to say something is more unique or less unique. Only
adjectives that admit of degree, adjectives about which we can legitimately say more or less, can be used in these two structures.

Notice, we could use a participle in the place of an ordinary adjective.


1) The highest peaks of the Himalayas are so tall that there is insufficient oxygen to
support life at the peaks.
2) The appeal of the 1942 film Casablanca is so enduring that, ever since its
release, it has been named as one of the greatest films of all times.
3) Glenn Gould was so devoted to recreating flawlessly the intentions of the
composers that he foreswore live performance entirely and chose to perform only in
the pristine conditions of the recording studio for the last 18 years of his life.
4) The planets Venus and Jupiter are so bright as to outshine easily the stars of
the night time sky.

5) Critics suggest that Johnson became so concerned with the American


involvement in Vietnam as toneglect entirely the social programs for which he had
been elected.
Notice, in both #3 and #5, the adjectives place is taken by long participial phrases:
expect to see similar constructions on the GMAT Sentence Correction.

También podría gustarte